You are on page 1of 72

ATLS PRE TEST 1 – 4 QUESTIONS AND CORRECT

ANSWERS 320+ QUESTIONS AND ANSWERS|


GRADED A
Which of the following physical findings suggest a cause of hypotension other than
spinal cord injury?
A. Prispism
B. Bradycardia
C. Diaphragmatic breathing
D. Presence of deep tendon reflexes
E. Ability to flex forearms but not extend them -ANSWER- D. Presence of deep
tendon reflexes. Spinal shock refers to loss of muscle toe (flaccidty) and loss of
reflexes.

The primary indication for transferring A patient to a higher level trauma center is:
A. Unavailibility of surgeon or operating staff
B. Multiple system injuries, including severe head injury
C. Resource limitations as determined by the transferring doctor
D. Resource limitations as determined by the hospital administration
E. Widened mediastinum on chest x-ray following blunt trauma -ANSWER- C.
Resource limitations as determined by the transferring doctor (MÅ SJEKKES)

A young man sustains a rifle wound to the mid-abdomen. He is brought promptly


to the ED by prehospital personnel. His skin is cool and diaphoretic, and his
systolic blood pressure is 58mmHg. Warmed crystalloid fluids are initiated without
improvement in his vital signs. The next, most appropriate, step is to perform:
A. a laparotomy
B. An abdominal CT-scan
C. Diagnostic laparoscopy
D. Abdominal ultrasonography
E. A diagnostic peritoneal lavage -ANSWER- A. Laparotomy because of
hemodynamic abnormality

A 42-year-old man is trapped from the waist down beneath his overturned tractor
for several hours before medical assistance arrives. He is awake and alert until just
before arriving in the ED. He is now unconscious and responds only to painful
stimuli by moaning. His pupils are 3mm in diameter and symmetrically reactive to
light. Prehospital personnel indicate that they have not seen the patient move either
of his lower extremities. On examination in the ED, no movement of his lower
extremities are detected, even in response to painful stimuli. The most likely cause
for this finding is:
A. An epidural hematoma
B. A pelvic fracture
C. Central cord syndrome
D. Intracerebral hemorrhage
E. Bilateral compartment syndrome -ANSWER- MÅ SJEKKES

A 6-year-o boy is struck by an automobile and brought to the ED. He is lethargic,


but withdraws purposefully from painful stimuli. His blood pressure is 90mmHg
systolic, heart rate 140 beats per minute and his respiratory rate is 36 breaths per
minute. The preferred route of venous access in this patient is:
A. Percutaneous femoral vein cannulation
B. Cutdown on the saphenous vein at the ankle
C. Intraosseous catheter placement in the proximal tibia
D. Percutaneous peripheral veins in the upper extremities
E. Central venous access via the subclavian or internal jugular vein -ANSWER- D.
Percutaneous peripheral veins in the upper extremities

A young man sustains a gunshot wound to the abdomen and is brought promptly to
the ED by prehospital personnel. His skin is cool and diaphoretic, and he is
confused. His pulse is thready and his femoral pulse is only weakly palpable. The
definitive treatment in managing this patient is to:
A. Administer O-negative blood
B. Apply external warming devices
C. Control internal hemorrhage operatively
D. Apply a pneumatic antishock garment (PASG)
E. Infuse large volumes of intravenous crystalloid solutions. -ANSWER- C.
Control internal hemorrhage operatively

Regarding shock in the child, which of the following is FALSE?


A. Vital signs are age-related
B. Children have greater physiologic reserves than do adults
C. Tachycardia is the primary physiologic response to hypovolemia
D. The absolute volume of blood loss required to produce shock is the same as in
adults
E. An initial fluid bolus for resuscitation should approximate 20ml/kg Ringers
Lactate -ANSWER- D. The absolute volume of blood loss required to produce
shock is the same as in adults
A 33-year-old man is struck by a car travelling at 56km/h (35mph). He has obvious
fractures of the left tibia near the knee, pain in the pelvic area, and severe dyspnea.
His heart rate is 182 beats per minute, and his respiratory rate is 48 breaths per
minute with no breath sounds heard in the left chest. A tension pneumothorax is
relieved by immediate needle decompression and tube thoracostomy.
Subsequently, his heart rate decreases to 144 beats per minute, his respirartory rate
decreases to 36 breaths per minute and his blood pressure is 81/53 mmHg.
Warmed Ringers lactate is adminstered intravenously. The next priority should be
to:
A. Perform external fixation of the pelvis
B. Obtain abdominal and pelvic CT-scans
C. Perform arterial embolization of the pelvic vessel
D. Perform diagnostic peritoneal lavage or FAST
E. Perform a urethrogram and cystogram -ANSWER- D. Perform diagnostic
peritoneal lavage or FAST

A 42-year-old man, injured in a motor vehicle crash, suffers a closed head injury,
multiple palpable left rib fractures, and bilateral femur fractures. He is intubated
orotracheally without difficulty. Initially, his ventilations are Reasily assisted with
a bag-mask device. It becomes more difficult to ventilate the patient over the next
5 minutes, and his hemoglobin oxygen saturation level decreases from 98% to
89%. The most appropriate next step is to:
A. Obtain a chest x-ray
B. Decrease the tidal volume
C. Decrease PEEP
D. Increase the rate of assisted ventilations
E. Perform needle decompression of the left chest. -ANSWER- A. Obtain a chest
x-ray (MÅ SJEKKES)

A 30-year-old man sustains a severely comminuted, open, distal right femur


fracture in a motorcycle crash. The wound is actively bleeding. Normal sensation
is present over the lateral aspect of the foot but decreased over the medial foot and
great toe. Normal motion of the foot is observed. Dorsalis pedis and posterior tibial
pulses are easily palpable on the left, but heard only by Doppler on the right.
Immediate efforts to improve circulation to the injured extremity should involve:
A. Immediate angiography
B. Tamponade of the wound with a pressure dressing
C. Wound exploration and removal of bony fragments
D. Realignment of the fracture segments with a traction splint
E. Fasciotomy of all four compartments in the lower extremity -ANSWER- B.
Tamponade of the wound with a pressure dressing

An 18-yeard-old, unhelmeted motorcyclist is brought by ambulance to the ED


following a crash. He had decreased level of consciousness at the scene, but then
was alert and conversational during transportation. Now his GCS is only 11.
Which of the following statements is TRUE?
A. Cerebral perfusion is intact
B. Intravascular volume status is normal
C. The patient is in a postictal state
D. Intra-abdominal visceral injury is unlikely
E. The patient probably has an acute epidural hematoma -ANSWER- E. The
patient probably has an acute epidural hematoma

A previously healthy, 70kg (175 pound) man suffers an estimated acute blood loss
of two liters. Which one of the following statements apply to this patient?
A. His pulse pressure will be widened
B. His urinary output will be at the lower limits of normal
C. He will have tachycardia, but no change in systolic blood pressure
D. His systolic blood pressure will be decreased with a narrowed, pulse pressure
E. His systolic blood pressure will be maintained with an elevated diastolic
pressure. -ANSWER- E. His systolic blood pressure will be maintained with an
elevated diastolic pressure.

The physioclogic hypervolemia of pregnancy has clinical significance in the


management of the severely injured gravid woman by
A. Reducing the need for blood transfusion
B. Increasing the risk of pulmonary edema
C. Complicating the management of closed head injury
D. Increasing the volume of blood loss to produce shock/maternal hypotension
E. Reducing the volume of crystalloid required for resuscitation -ANSWER- D.
Increasing the volume of blood loss to produce shock

A 17-year-old helmeted motorcyclist loses consciousness when he is struck broad


side by an automobile at an intersection. He arrives in the ED with a blood
pressure of 140/92, pulse rate 88 beats per minute, a respiratory rate of 18 breaths
per minue, and a GCS of 7. Appropriate initial immobilization of this patient
should include a semi-rigid cervical collar and:
A. A scoop stretcher
B. A long spine board
C. A short spine board
D. Cervical traction tongs
E. Pneumatic antishock garment -ANSWER- B. A long spine board

During an altercation, a 36-year-old man sustains a gunshot wound above the


nipple line on the right, with an exit wound posteriorly above the scapula on the
right. He is transported by ambulance to a community hospital. He is
endotracheally intubated, close tube thoracostomy is performed, and 2 liters
Ringers lactate solution are infused via 2 large-caliber IV´s. His blood pressure
now is 60/0mmHg, heart rate is 160 beats per minute, and respiratory rate is 14
breaths per minute (ventilated with 100% O2). The most appropriate next step in
managin this patient is:
A. Laparotomy
B. Diagnostic peritoneal lavage
C. Arterial blood gas determination
D. Administer packed red blood cells
E. Chest X-ray to confirm tube placement -ANSWER- E. Chest X-ray to confirm
tube placement

Abscence of breath sounds and dullness to percussion over the left hemithorax are
findings best explained by:
A. Left hemothorax
B. Cardiac contusion
C. Left simple pneumothorax
D. Left diaphragmatic rupture
E. Right tension pneumothorax -ANSWER- A. Left hemothorax

A 23-year-old man is brought immediately to the ED from the hospitals parking lot
where he was shot in the lower abdomen. Examination reveals a single bullet
wound. He is breathing and has a thready pulse. However, he is unconsious and
has no detectable blood pressure. Optimale immediate management is to:
A. Perform a diagnostic peritoneal lavage
B. Initiate infusion of packed red blood cells
C. Insert a nasogastric tube and urinary catheter
D. Transfer the patient to the operating room, while initiating fluid therapy
E. Initiate fluid therapy to return his blood pressure to normotensive -ANSWER-
D. Transfer the patient to the operating room, while initiating fluid therapy

A teen-aged bicycle rider is hit by a truck traveling at high speed. In the ED, she is
actively bleeding from open fractures of her legs, and has abrasions on her chest
and abdominal wall. Her blood pressure is 80/50 mmHg, heart rate is 140 beats per
minute, respiratory rate is 8 breaths per minute, and GCS score is 6. The first step
in managing this patient is to:
A. Obtain a lateral cervical spine x-ray
B. Insert av central venous pressure line
C. Adminster 2 liters of crystalloid solution
D. Perform endotracheal intubation and ventilation
E. Apply a pneumatic antishock garment (PASG) and inflate the leg compartments.
-ANSWER- D. Perform endotracheal intubation and ventilation

An 8-year-old boy falls 4,5 meters (15 feet) from a tree and is brought to the ED by
his family. His vital signs are normal, but he complains of left upper quadrant pain.
An abdominal CT-scan reveals a moderately severe laceration of the spleen. The
receiving institution does not have 24-hour-a-day operating room capabilities. The
most appropriate management of this patient would be to
A. Type and crossmatch for blood
B. Request consultation of a pediatrician
C. Transfer the patient to a trauma center
D. Admit the patient to the intensive care unit
E. Prepare the patient for surgery the next day -ANSWER- D. Admit the patient to
the intensive care unit

Which of the following statements regarding injury to the central nervous system
in children is TRUE?
A. Children suffer spinal cord injury without x-ray abnormality more commonly
than adults.
B. An infant with a traumatic brain injury may become hypotensive from cerebral
edema
C. Initial therapy for the child with traumatic brain injury includes the
administration of methylprednisolone intravenously
D. Children have more focal mass lesions as a result for traumatic brain injury
when compared to adults.
E. Young children are less tolerant of expanding intracranial mass lesions than
adults -ANSWER- A. Children suffer spinal cord injury without x-ray abnormality
more commonly than adults.

A 17-year-old helmeted motorcyclist is struck broadside by an automobile at an


intersection. He is unconscious at the scene with a blood pressure of 140/90mmHg,
heart rate of 90 beats per minute, and respiratory rate of 22 breaths per minute. His
respirations are sonorous and deep. His GCS score is 6. Immobilization of the
entire patient may include the use of all the following EXCEPT:
A. Air splints
B. Bolstering devices
C. A long spine board
D. A scoop-style stretcher
E. A semi-rigid cervical collar -ANSWER- A. Air splints

Twenty-seven patients are seriously injured in an aircraft accident at a local airport.


The basic principle of triage should be to:
A. Treat the most severely injured patients first
B. Establish a field triage area directed by a doctor
C. Rapidly transport all patients to the nearest appropriate hospital
D. Treat the greatest number of patients in the shortest period of time
E. Produce the greatest number of survivors based on available resources -
ANSWER- E. Produce the greatest number of survivors based on available
resources

An electrician is eletrocuted by a downed power line after a thunderstorm. He


apparently made contact with the wire at the level of the right mid thigh. In the ED,
his vital signs are normal and no dysrythmia is noted on ECG. On examination,
there is an exit wound on the bottom of the right foot. His urine is positive for
blood by dipstick but not RBCs are seen microscopically. Initial management
should include:
A. Immediate angiography
B. Aggressive fluid infusion
C. Intravenous pyelography
D. Debridement of necrotic muscle
E. Admission to the ICU for observation -ANSWER- B. Aggressive fluid infusion
- suspected rhabdomyolyse

A young woman sustains a severe head injury as the result of a motor vehicular
crash. In the ED, her GCS is 6. Her blood pressure is 140/90 mmHg and her heart
rate 80 beats per minute. She is intubated and is being mechanically ventilated. Her
pupils are 3mm in size and equally reactive to light. There is no other apparent
injury. The most important principle to follow in early management of her head
injury is to:
A. Administer an osmotic diuretic
B. Prevent secondary brain injury
C. Agressively treat systemic hypertension
D. Reduce meatbolic requirements of the brain
E. Distinguish between intracranial hematoma and cerebral edema. -ANSWER- B.
Prevent secondary brain injury

To establish a diagnosis of shock,


A. Systolic blood pressure must be below 90mmHg
B. The presence of a closed head injury should be excluded
C. Acidosis should be present by arterial blood gas analysis
D. The patient must fail to respond to intravenous fluid infusion
E. Clinical evidence of inadequate organ perfusion must be present. -ANSWER- E.
Clinical evidence of inadequate organ perfusion must be present.

A 32-year-old is brought to the hospital unconscious with severe facial injuries and
noisy respirations after an automobile collision. In the ED, he has no apparent
injury to the anterior aspect of his neck. He suddenly becomes apneic, and
attempted ventilation with a face mask is unsuccessful. Examination of his mouth
reveals a large hematoma of the pharynx with loss of normal anatomic landmarks.
Initial management of his airways should be consist of:
A. Inserting an oropharyngeal airway
B. Inserting a nasopharyngeal airway
C. Performing a surgical cricothyroidotomy
D. Performing fiberoptic-guided nasotracheal intubation
E. Performin orotracheal intubation after obtaining a lateral c-spine x-ray -
ANSWER- A. Inserting an oropharyngeal airway

A 25-year-old woman is brought to the ED after a motor vehicle crash. She was
initially lucid at the scene and then developed a dilated pupil and contralateral
extremity weakness. In the ED, she is unconscious and has a GCS score of 6. The
initial management step for this patient should be to:
A. Obtain a CT-scan of the head
B. Administer decadron 20mg IV
C. Perform endotracheal intubation
D. Administer mannitol 1g/kg IV
E. Perform an emergency bone flap craniotomy on the side of the dilated pupil. -
ANSWER- C. Perform endotracheal intubation

A contraindication to nasogastric intubation is the presence of a:


A. Gastric perforation
B. Diaphragmatic rupture
C. Open depressed skull fracture
D. Fracture of the cervical spine
E. Fracture of the cribiform plate -ANSWER- E. Fracture of the cribiform plate

An 8-year-old girl is an unrestrained passenger in a vehicle struck from behind. In


the ED, her blood pressure is 80/60mmHg, heart rate is 80 beats per minute, and
respiratory rate is 16 breaths per minute. Her GCS score is 14. She complains that
her legs feel "funny and wont move right". However, her spine x-rays do not show
a fracture or dislocation. A spinal cord injury in this child:
A. Is most likely a central cord syndrome
B. Must be diagnosed by magnetic resonance imaging
C. Can be excluded by obtaining a CT-scan of the entire spine
D. May exist in the abscence of objective findings on x-ray studies
E. Is unlikely because of the incomplete calcification of the vertebral bodies. -
ANSWER- D. May exist in the abscence of objective findings on x-ray studies

Immediate chest tube insertion is indicated for which of the following conditions?
A. Pneumothorax
B. Pneumomediastinum
C. Massive hemothorax
D. Diaphragmatic rupture
E. Subcutaneous emphysema -ANSWER- C. Massive hemothorax

Cardiac tamponade after trauma:


A. Is seldom life-threating
B. Can be excluded by an upright, AP chest x-ray
C. Can be confused with a tension pneumthorax
D. Causes a fall in systolic pressure of > 15mmHg with expiration
E. Most commonly occurs after blunt injury to the anterior chest wall -ANSWER-
C. Can be confused with a tension pneumthorax

A 22-year-old man is brought to the hospital after crashing his motorcycle into a
telephone pole. He is unconscious and in profound shock. He has no open wounds
or obvious fractures. The cause of his shock is MOST LIKELY caused by:
A. A subdural hematoma
B. An epidural hematoma
C. A transected lumbar spinal cord
D. A basilar skull fracture
E. Hemorrhage into the chest or abdomen -ANSWER- E. Hemorrhage into the
chest or abdomen
Which of the following statements is FALSE concerning Rh-isoimmunization in
the pregnant trauma patient?
A. It occurs in blunt or penetrating abdominal trauma
B. Minor degrees of fetomaternal hemorrhage produce it
C. A negative Kleihauer-Betke test excludes Rh-Isoimmunzation
D. This is not a problem in the traumatized Rh-positive pregnant patient
E. Initiation of Rh-immunoglobulin therapy does not require proof of fetomaternal
hemorrhage -ANSWER- C. A negative Kleihauer-Betke test excludes Rh-
Isoimmunzation

All of the following signs on the chest x-ray of a blunt injury victim may suggest
aortic rupture EXCEPT:
A. Mediastinal emphysema
B. Presence of a "pleural cap"
C. Obliteration of the aortic knob
D. Deviation of the trachea to the right
E. Depression of the left mainstem bronchus -ANSWER- A. Mediastinal
emphysema

Early central venous pressure monitoring during fluid resusciation in the ED has
the greatest utility in a:
A. Patient with a splenic laceration
B. Patient with a inhalation injury
C. 6 year-old child with a pelvic fracture
D. Patient with a severe cardiac contusion
E. 24-year-old man with a massive hemothorax -ANSWER- D. Patient with a
severe cardiac contusion

A cross-table lateral x-ray of the cervical spine:


A. must precede endotracheal intubation
B. excludes serious cervical spine injury
C. Is an essential part of the primary survey
D. Is not necessary for unconscious patients with penetrating cervical injuries
E. Is unacceptable unless 7 cervical vertebrae and the C-7 to T-1 relationship are
visualized. -ANSWER- E. Is unacceptable unless 7 cervical vertebrae and the C-7
to T-1 relationship are visualized.

A 24-year old man sustains multiple fractured ribs bilaterally as a result of being
crushed in a press at a plywood factory. Examination in the ED reveals a flail
segment of the patients thorax. Primary resuscitation includes high-flow oxygen
administration via a nonrebreathing mask, and initiation of Ringers lactate
solution. The patient exhibits progressive confusion, cyanosis and tachypnea.
Management at this time should consist of:
A. Intravenous sedation
B. External stabilization of the chest wall
C. Increasing the FIO2 in the inspired gas
D. Intercostal nerve blocks for pain relief
E. Endotracheal intubation and mechanical ventilation. -ANSWER- E.
Endotracheal intubation and mechanical ventilation

Which of the following statements regarding patients with thoracic spine injuries is
TRUE?
A. Log-rolling may be destabilizing to fractures from T12 to L1
B. Adequate immobilization can be accomplished with the scoop stretcher
C. Spinal cord injury below T10 usually spares bowel and bladder function
D. Hyperflexion fractures in the upper thoracic spine are inherently unstable
E. These patients rarely present with neurogenic shock in association with cord
injury. -ANSWER- A. Log-rolling may be destabilizing to fractures from T12 to
L1

During resuscitation, which one of the following is the most reliable as a guide to
volume replacement?
A. Heart rate
B. Hematocrit
C. Blood pressure
D. Urinary output
E. Jugular venous pressure -ANSWER- D. Urinary out

A 24-year-old woman passenger in an automobile strikes the wind screen with her
face during a head-on collision. In the ED, she is talking and has marked facial
edema and crepitus. The highest priority should be given to:
A. Lateral c-spine x-ray
B. Upper airway protection
C. Carotid pulse assessment
D. Management of blod loss
E. Determination of associated injuries -ANSWER- B. Upper airway protection

The driver of a single car crash is orotracheally intubated in the field by prehospital
personnel after they identify a closed head injury and determine that the patient is
unable to protect his airway. In the ED, the patient demonstrate decorticate
posturing bilaterally. He is being ventilated with a bag-valve device, but his breath
sounds are absent in the left hemithorax. His blood pressure is 160/80mmHg, heart
rate is 70 beats per minute, and the pulse oximeter displays a hemoglobin oxygen
saturation of 96%. The next step in assessing and managing this patient should be
to:
A. Determine the arterial blood gases
B. Obtain a lateral cervical spine x-ray
C. Assess placement of the endotracheal tube
D. Perform needle decompression of the left chest
E. Insert a thoracostomy in the left hemithorax. -ANSWER- C. Assess placement
of the endotracheal tube

The response to catecholamines in an injured, hypovolemic pregnant woman can


be expected to result in:
A. Placental abruption
B. Fetal hypoxia and distress
C. Fetal/maternal dysrhytmia
D. Improved uterine blood flow
E. Increased maternal renal blood flow -ANSWER- B. Fetal hypoxia and distress

A 22-year-old man sustains a gunshot wound to the left chest and is transported to
a small community hospital at which surgical capabilites are not available. In the
ED, a chest tube is inserted and 700ml of blood is evacuted. The trauma center
accepts the patient in transfer. Just before the patient is placed in an ambulance for
transfer, his blood pressure decreases to 80/68mmHg and his heart rate increases to
136 beats per minute. The next step should be to:
A. Clamp the chest tube
B. Cancel the patients transfer
C. Perform an ED thoracotomy
D. Repeat the primary survey and proceed with transfer
E. Delay the transfer until the referring doctor can contact a thoracic surgeon. -
ANSWER- D. Repeat the primary survey and proceed with transfer

A young woman sustains a severe head injury as the result of a motor vehicular
crash. In the ED, her GCS is 6. Her blood pressure is 140/90 mmHg and her heart
rate 80 beats per minute. She is intubated and is being mechanically ventilated. Her
pupils are 3mm in size and equally reactive to light. There is no other apparent
injury. The most important principle to follow in early management of her head
injury is to:
A. Avoid hypotension
B. Prevent secondary brain injury
C. Agressively treat systemic hypertension
D. Reduce meatbolic requirements of the brain
E. Distinguish between intracranial hematoma and cerebral edema. -ANSWER- A.
Avoid hypotension

A 6-year-old boy walking across the street is struck by the front bumper of a sports
utility vehicle traveling at 32kph (20mph). Which one of the following statements
is TRUE?
A. A flail chest is probable
B. A symptomatic cardiac contusion is expected
C. A pulmonary contusion may be present in the absence of rib fractures
D. Transection of the thoracic aorta is more likely than in an adult patient
E. Rib fractures are commonly found in children with this mechanism of injury -
ANSWER- C. A pulmonary contusion may be present in the absence of rib
fractures

A 39-year-old man is admitted to the ED after an automobile collision. He is


cyanotic, has insufficient respiratory effort, and has a GCS score of 6. His full
beard makes it difficult to fit the oxygen facemask to his face. The most
appropriate next step is to:
A. Perform a surgical cricothyroidotomy
B. Attempt nasotracheal intubation
C. Ventilate him with a bag-mask device until c-spine injury can be excluded
D. Attempt orotracheal intubation using 2 people and inline stabilization of the
cervical spine
E. Ventilate the patient with a bag-mask device until his beard can be shaved for a
better mask fit. -ANSWER- D. Attempt orotracheal intubation using 2 people and
inline stabilization of the cervical spine

A patient is brought to the ED 20 minutes after a motor vehicle crash. He is


conscious and there is no obvious external trauma. He arrives at the hospital
completely immobilized on a long spine board. His blood pressure is 60/40mmHg
and his heart rate is 70 beats per minute. His skin is warm. Which one of the
following statements is true?
A. Vasoactive medications have no role in the patients management
B. The hypotension should be managed with volume resuscitation alone
C. Flexion and extension views on the c-spine should be performed early
D. Occult abdominal visceral injuries can be excluded as a cause of hypotension
E. Flaccidity of the lower extremities and loss of deep tendon reflexes are
expected. -ANSWER- E. Flaccidity of the lower extremities and loss of deep
tendon reflexes are expected.

The following are contraindications for tetanus toxoid administration:


A. History of neurological reaction or severe hypersensitivity to the product
B. Local side effects
C. Muscular spasm
D. Pregnancy
E. All of the above -ANSWER- A. History of neurological reaction or severe
hypersensitivity to the product

After being involved in a motor vehicle crash, a 25-year-old man is brought to a


hospital with a general surgeon on duty. He has a GCS of 13 and complains of
abdominal pain. His blood pressure was 80mmHg systolic by palpation on arrival
at the hospital, but increases to 110/70 wit the administration of 2L of IV-fluid. His
heart rate remains 120 beats per minute. CT shows an aortic injury and splenic
laceration with free abdominal fluid. His blood pressure falls to 70mmHg after CT.
The next step is to
A. Contrast angiography
B. Transfer to a higher level trauma center
C. Exploratory laparotomy
D. Transfuse packed red blood cells
E. Transesophageal echocardiography -ANSWER- C. Exploratory laparotomy

Which one of the following statements regarding abdominal trauma in the pregnant
patient is TRUE?
A. The fetus is in jeopardy only with major abdominal trauma
B. Leakage of amniotic fluid is an indication for hospital admission
C. Indications for peritoneal lavage are different from those in the non-pregnant
patient
D. Penetration of an abdominal hollow viscus is mor common late than in early
pregnancy
E. The secondary survey follows a different pattern from that of the non-pregnant
patient -ANSWER- B. Leakage of amniotic fluid is an indication for hospital
admission

All of the following are indicators of inhalation injury EXCEPT:


A. Singeing of the eyebrows and nasal vibrissae
B. Carboxyhemoglobin level >4%
C. Carbon deposits in the mouth or nose and carbonaceous sputum
D. Hoarseness
E. Face or neck burns -ANSWER- B. Carboxyhemoglobin level >4% (grensen er
10%)

A 32-year-old man right leg is trapped beneath his overturned car for nearly 2
hours before he is extricated. On arrival in the ED, his right lower extremity is
cool, mottled, insensate, and motionless. Despite normal vital signs, pulses cannot
be palpated inferior to the femoral artery, and the muscles of the lower extremity
are firm and hard. During initial management of this patient, which of the
following is most likely to improve chances for limb salvage?
A. Apply skeletal traction
B. Administering anticoagulant drugs
C. Adminstering thrombolytic therapy
D. Perform right lower extremity fasciotomy
E. Immediately transferring the patient to a trauma center. -ANSWER- D. Perform
right lower extremity fasciotomy

A patient arrives in the ED after being beaten about the head and face with a
wooden club. He is comatose and has a palpable depressed skull fracture. His face
is swollen and ecchymotic. He has gurgling respirations and vomitus on his face
and clothing. The most appropriate step after providing supplemental oxygen and
elevating his jaw is to:
A. Request a CT-scan
B. Insert a gastric tube
C. Suction of the oropharynx
D. Obtain a lateral cervical spine x-ray
E. Ventilate the patient with a bag-mask -ANSWER- C. Suction of the oropharynx

A 64-year-old man, involved in a high-speed car crash, is resuscitated initially in a


small hospital with limited resources. He has a closed head injury with a GCS
score of 13. He has a widened mediastinum on chest x-ray with fractures of left
ribs 2 through 4 but no pneumothorax. After infusing 2 liters of crystalloid
solution, his blood pressure is 100/74, heart rate is 110 beats per minute, and
respiratory rate is 18 breaths per minute. He has gross hematuria and a pelvic
fracture. You decide to transfer this patient to a facility capable of providing a
higher level of care. The facility is 128km (80 miles) away. Before transfer, you
should first:
A. Intubate the patient
B. Perform diagnostic peritoneal lavage
C. Apply the pneumatic antishock garment
D. Call the receiving hospital and speak to the surgeon on call
E. Discuss the advisability of transfer with the patients family. -ANSWER- D. Call
the receiving hospital and speak to the surgeon on call

During the third trimester of pregnancy, all of the following changes occur
normally, EXCEPT a:
A. Decrease in PaCO2
B. Decrease in the leukocyte count
C. Reduce gastric emptying rate
D. Diminished residual lung volume
E. Diminished pelvic ligament tension -ANSWER- B. Decrease in the leukocyte
count

In managing the head-injury patient, the most important initial step is to:
A. Secure the airway
B. Obtain c-spine film
C. Support the circulation
D. Control scalp hemorrhage
E. Determine GCS score -ANSWER- A. Secure the airway

The first maneuver to improve oxygenation after chest injury is:


A. Intubate the patient
B. Assess arterial blood gases
C. Administer supplemental oxygen
D. Ascertain the need for a chest tube
E. Obtain a chest x-ray -ANSWER- C. Administer supplemental oxygen

A 25-year-old man, injured in a motor vehicular crash, is admitted to the ED. His
pupils react sluggishly and his eyes open to painful stimuli only. He does not
follow commands, but he does moan periodically. His right arm is deformed and
does not respond to painful stimulus; however, his left hand reaches purposefully
toward the painful stimulus. Both legs are stiffly extended. His GCS score is:
A. 7
B. 8
C. 9
D. 10
E. 11 -ANSWER- 9
A 20-year-old woman, at 32 weeks gestation, is stabbed in the upper right chest. In
the ED, her blood pressure is 80/60mmHg. She is gasping for breath, extremely
anxious, and yelling for help. Breath sounds are diminished in the right chest. The
most appropriate first step is to:
A. perform tracheal intubation
B. Insert an oropharyngeal airway
C. Perform needle decompression of the right chest
D. Manually displace the gravid uterus to the left side of the abdomen
E. Initiate 2 large-caliber peripheral IV lines and crystalloid infusion -ANSWER-
C. Perform needle decompression of the right chest

Which one of the following findings in an adult should prompt immediate


management during primary survey?
A. Distended abdomen
B. GCS of 11
C. Temperature of 36,5
D. Heart rate of 120 beats per minute
E. Respiratory rate of 40 breaths per minute -ANSWER- E. Respiratory rate of 40
breaths per minute

A trauma patient present to your emergency department with inspiratory stridor


and a suspected C-spine injury. Oxygen saturation is 88% on high-flow oxygen via
a nonrebreathing mask. The most appropriate next step is to:
A. Apply cervical traction
B. Perform immediate tracheostomy
C. Insert bilateral thoracostomy tubes
D. Maintain 100% oxygen and obtain immediate c-spine x-rays
E. Maintain inline immobilization and establish a definitive airway -ANSWER- E.
Maintain inline immobilization and establish a definitive airway

When apply the Rule of Nines to infants,


A. It is not reliable
B. The body is proportionally larger in infants than in adults
C. The head is proportionally larger in infants than in adults
D. The legs are proportionally larger in infants than in adults
E. The arms are proportionally larger in infants than in adults -ANSWER- C. The
head is proportionally larger in infants than in adults

A 60-year-old man sustains a stab wound to the right posterior flank. Witnesses
state the weapon was a small knife. His heart rate is 90 beats per minute, blood
pressure is 128/72mmHg and respiratory rate is 24 breaths per minute. The most
appropriate action to take at this time is to:
A. Perform a colonoscopy
B. Perform a barium enema
C. Perform an intravenous pyelogram
D. Perform serial physical examinations
E. Suture repair the wound and outpatient follow up -ANSWER- D. Perform serial
physical examinations

The following are criteria for transfer to a burn center, EXCEPT for:
A. Partial-thickness and full-thickness burns on greater than 10% of the BSA
B. Any full-thickness burn
C. Partial-thickness and full-thickness burn involving the face, hands, feet,
genitalia, perineum and skin overlying major joints
D. Elevated central venous pressure
E. Inhalation injury -ANSWER- D. Elevated central venous pressure

Systolic blood pressure starts to decrease in which class of hemorrhage?


A. Class 0
B. Class 1
C. Class 2
D. Class 3
E. Class 4 -ANSWER- C. Class 2

A 7-year-old boy is brought to the ED by his parents several minutes after he fell
through a window. He is bleeding profusely from a 6-cm wound of his medial right
thigh. Immediate management of the wound should consist of:
A. Application of a torniquet
B. Direct pressure on the wound
C. Packing the wound with gauze
D. Direct pressure on the femoral artery at the groin
E. Debridement of devitalized tissue -ANSWER- B. Direct pressure on the wound

For the patient with severe traumatic brain injury, profound hypocarbia should be
avoided to prevent:
A. Respiratory alkalosis
B. Metabolic acidosis
C. Cerebral vasoconstriction with diminished perfusion
D. Neurogenic pulmonary edema
E. Shift of the oxyhemoglobin dissociation curve. -ANSWER- C. Cerebral
vasoconstriction with diminished perfusion

A 33-year-old woman is involved in a head-on motor vehicle crash. It took 30


minutes to extricate her from the car. Upon arrival in the ED, her heart rate is 120
beats per minute, BP is 90/70mmHg, respiratory rate is 16 breaths per minute, and
GCS is 15. Examination reveals bilaterally equal breath sounds, anterior chest wall
ecchymosis, and distended neck veins. Her abdomen is flat, soft, and not tender.
Her pelvis is stable. Palpable distal pulses are found in all 4 extremities. Of the
following, the most likely diagnosis is:
A. hemorrhagic shock
B. Cardiac tamponade
C. Massive hemothorax
D. Tension pneumothorax
E. Diaphragmatic rupture. -ANSWER- B. Cardiac tamponade

A hemodynamically normal 10-year-old girls is admitted to the pediatric intensive


care unit for observation after a grade III (moderately severe) splenic injury has
been confirmed by CT. Which of the following mandates prompt laparotomy?
A. Serum amylase of 200
B. Leukocyte count of 14,000
C. Extraperitoneal bladder rupture
D. Free peritoneal air demonstrated on follow up CT
E. A fall in the hemoglobin level from 12g/dl to 8g/dl over 24 hours -ANSWER-
D. Free peritoneal air demonstrated on follow up CT

A 40-year-old woman restrained driver is transported to the ED in full spinal


immobilization. She is hemodynamically stable and found to be paraplegic at the
level of T10. Neurologic examination also determines that there is loss of pain and
temperature sensation with preservation of proprioception and vibration. These
finding are consistent with the diagnosis of :
A. Central cord syndrome
B. Spinal shock syndrome
C. Anterior cord syndrome
D. Complete cord syndrome
E. Brown-Sequard syndrome -ANSWER- C. Anterior cord syndrome

Hemorrhage of 20% of the patients blood volume is associated usually with:


A. Oliguria
B. Confusion
C. Hypotension
D. Tachycardia
E. Blood transfusion requirement -ANSWER- D. Tachycardia

Which of the follow statements concerning intraosseous infusion is TRUE?


A. Only crystalloid solutions may be safely infused through the needle
B. Aspiration of bone marrow confirms appropriate positioning of the needle
C. Intraosseous infusion is the preferred route for volume resuscitation in small
children
D. Intraosseous infusion may be utilized indefinitely
E. Swelling in the soft tissues around the intraosseous site is not a reason to
discontinue infusion. -ANSWER- B. Aspiration of bone marrow confirms
appropriate positioning of the needle

The most important, immediate step in the management of an open pneumothorax


is:
A. endotracheal intubation
B. Operation to close the wound
C. Placing a chest tube through the chest wound
D. Placement of an occlusive dressing over the wound
E. Initiation of 2 large-caliber IVs with crystalloid solution -ANSWER- D.
Placement of an occlusive dressing over the wound

Which one of the following situations requires Rh immunoglobulin administration


to an injured woman?
A. Negative pregnancy test, Rh negative, and torso trauma
B. Positive pregnancy test, Rh positive, and has torso trauma
C. Positive pregnancy test, Rh negative, and has torso trauma
D. Positive pregnancy test, Rh positive, and has an isolated wrist fracture
E. Positive pregnancy test, Rh negative, and has an isolated wrist fracture -
ANSWER- C. Positive pregnancy test, Rh negative, and has torso trauma

A 22-year-old man is hypotensive and tachycardic after a shotgun wound to the left
shoulder. His BP is initially 80/40mmHg. After 2 liters of crystalloid solution his
blood pressure increases to 122/84mmHg. His heart rate is now 100 beats per
minute and his respiratory rate is 28 breaths per minute. His breath sounds are
decreased in the left hemithorax, and after initial IV fluid resusciation, a closed
tube thoracostomy is performed for decreased left breath sound with the return of
small amount of blood and no air leak. After chest tube insertion, the most
appropriate next step is to:
A. Reexamine the chest
B. Perform an aortogram
C. Obtain a CT-scan of the chest
D. Obtain arterial blood gas analyses
E. Perform transesophageal echocardiography -ANSWER- A. Reexamine the chest

A construction worker falls two stories from a building and sustain bilateral
calcaneal fractures. In the ED, he is alert, vital signs are normal, and he is
complaining of severe pain in both heels and his lower back. Lower extremity
pulses are strong and there is no deformity. The suspected diagnosis is most likely
to be confirmed by:
A. Angiography
B. Compartment pressures
C. Retrograde urethrogram
D. Doppler-ultrasound studies
E. Complete spine x-ray series -ANSWER- E. Complete spine x-ray series

A 22-year-old female athlete is stabbed in her left chest at the third interspace in
the anterior axillary line. On admission to the ED and 15 minutes after the incident,
she is awake and alert. Her heart rate is 100 beats per minute, BP 80/60mmHg, and
respiratory rate is 20 breaths per minute. A chest x-ray reveals a large left
hemithorax. A left chest tube is placed with an immediate return of 1600ml of
blood. The next management step for this patient is:
A. perform a thoracoscopy
B. Perform an arch aortogram
C. Insert a second left chest tube
D. Prepare for an exploratory thoracotomy
E. Perform a chest CT -ANSWER- D. Prepare for an exploratory thoracotomy

A 56-year-old man is thrown violently against the steering wheel of his truck
during a motor vehicle crash. On arrival in the ED he is diaphoretic and
complaining of chest pain. His BP is 60/40mmHg and his respiratory rate is 40
breaths per minute. Which of the following best differentiates cardiac tamponade
from tension pneumothorax as the cause of his hypotension?
A. Tachycardia
B. Pulse volume
C. Breath sounds
D. Pulse pressure
E. Jugular venous pressure -ANSWER- C. Breath sounds
All of the following are true of the Mallampati classification EXCEPT:
A. Class IV is the easiest intubation, while Class 1 is the most difficult
B. It helps assess for difficult intubations
C. It is part of the LEMON assessment
D. It comprises a visual assessment of the distance from the tongue base to the roof
of the mouth, and therefore the amount of space there is to work
E. A poor Mallampati score is associated with a higher incidens of obstructive
sleep apnea. -ANSWER- A. Class IV is the easiest intubation, while Class 1 is the
most difficult

A 23-year-old man sustains three stab wounds to the upper right chest during an
altercation and is brought by ambulance to a hospital that has full surgical
capabilities. His wounds are all above the nipple. He is endotracheally intubates,
closed tube thoracostomy is performed, and 2 liters of crystalloid solution are
infused through 2 large-caliber IVs. His BP i 60/0mmHg, heart rate is 160 beats
per minute, and respiratory rate is 14 breaths per minute (ventilated with 100%
O2). 1500ml of blood has drained from the right chest. The most appropriate next
step in managing this patient is to:
A. Perform FAST
B. Obtain a CT of the chest
C. Perform angiography
D. Urgently transfer the patient to the operating room
E. Immediately transfer the patient to a trauma center. -ANSWER- D. Urgently
transfer the patient to the operating room

Which of the following signs is LEAST reliable for diagnosing esophageal


intubation?
A. Symmetrical chest movement
B. End-tidal CO2 presence by colorimetry
C. Bilateral breath sounds
D. Oxygen saturation > 92%
E. ETT above carina on chest x-ray -ANSWER- E. ETT above carina on chest x-
ray

Which one of the following signs necessitates a definitive airway in severe trauma
patients?
A. Facial lacerations
B. Repeated vomiting
C. Severe maxillofacial fractures
D. Sternal fracture
E. GCS score of 12 -ANSWER- C. Severe maxillofacial fractures

Which one of the following statements is correct?


A. Cerebral contusion may coalesce to form an intracerebral hematoma
B. Epidural hematomas are usually seen in frontal region
C. Subdural hematomas are caused by injury to the middle meningeal artery
D. Subdural hematomas typically have a lenticular shape on CTscan
E. The associated brain damage is more severe in epidural hematomas. -
ANSWER- A. Cerebral contusion may coalesce to form an intracerebral hematoma

An 18 year old male is brought to the ED after having been shot. He has one bullet
wound just below the right clavicle and another just below the costal margin in the
right posterior axillary line. His BP is 110/60, HR is 90bpm, and RR is 34bpm.
After ensuring a patent airway and inserting 2 large caliber iv line, the next
appropriate step is to:
A. Obtain a chest x-ray
B. Adminster a bolus of additional iv fluid
C. Perform a laparotomy
D. Obtain abdominal CT-scan
E. Perform DPL -ANSWER- A. Obtain a chest x-ray

An 8 year old boy falls 4,5meters from a tree and is brought to the ED by his
familiy. His vital signs are normal, but he complains of left upper quadrant pain.
An abdominal CT scan reveals a moderately severe laceration of the spleen. The
receiving institution does not have 24 hour a day operating room capabilities. The
most appropriate management of this patient would be:
A. Type and crossmatch for blood
B. Request consultation of a pediatrician
C. Transfer the patient to a trauma center
D. Admit the patient to the ICU
E. Prepare the patient for surgery the next day -ANSWER- D. Admit the patient to
the ICU

A construction worker falls from a scaffold and is transferred to the ED. His HR is
124 bpm and BP is 85/60mmHg. He complains of lower abdominal pain. After
assessing the airway and chest, immobilizing the c-spine and initiating fluid
resuscitation, the next step is to perform
A. FAST
B. Detailed neurological exam
C. Rectal exam
D. Cervical c-spine x-ray
E. Urethral catheterization. -ANSWER- A. FAST

A 22 year old male sustains a shotgun wound to the left shoulder and chest at close
range. His BP is 80/40mmHg and his HR is 130bpm. After 2 liters of crystalloid
solution are rapidly infused, his BP increases to 122/84, and HR decreases to
100bpm. He is tachypneic with RR of 28. On physical examination, his breath
sounds are decreased at the left upper chest with dullness on percussion. A large
caliber (36 french) tube thoracostomy is inserted in the fifth intercostal space with
the return of 200ml of blood and no air leak. The most appropriate next step is to:
A. insert a foley catheter
B. Begin to transfuse o-negative blood
C. Perform thoracotomy
D. Obtain a CT-scan of chest and abdomen
E. Repeat the physical examination of the chest -ANSWER- E. Repeat the physical
examination of the chest

Which one of the following statements concerning spine and spinal cord trauma is
true?
A. A normal lateral c-spine film excludes injury
B. A vertebral injury is unlikely in the absence of physical findings of a cord injury
C. A patient with a suspected injury requires immobilization on a short spine
D. Diaphragmatic breathing in an unconscious patient who has fallen is a sign of
spine injury
E. Determination of whether a spinal cord lesion is complete or incomplete must
be made in the primary survey -ANSWER- D. Diaphragmatic breathing in an
unconscious patient who has fallen is a sign of spine injury

A 20 year old athlete is involved in a motorcycle crash. When he arrives in the ED,
he shouts that he cannot move his legs. On physical examination, there are noe
abnormalities of the chest, abdomen or pelvis. The patient has no sensation in his
legs and cannot move them, but his arms are moving. The patients RR is 28 bpm,
HR is 88bpm and BP is 80/60mmHg. He is pale and sweaty. What is the most
likely cause of this condition?
A. Neurogenic shock
B. Cardiogenic shock
C. Abdominal hemorrhage
D. Myocardial contusion
E. Hyperthermia. -ANSWER- A. Neurogenic shock
A 28 year old male is brought to the ED. He was involved in a fight in which he
was beaten with a wooden stick. His chest shows multiple severe bruises. His
airway is clear, RR is 22, HR is 126 and systolic BP is 90mmHg. Which one of the
following should be performed during the primary survey?
A. GCS
B. Cervical spine x-ray
C. TT-administration
D. Blood alcohol level
E. Rectal exam -ANSWER- A. GCS

Which one of the following statements is true regarding access in pediatric


resuscitation?
A. Intraosseous access should only be considered after five percutaneous attempts
B. Cut down at the ankle is a preferred initial access technique
C. Blood transfusion can be delievered through an intraosseous access
D. Internal jugular cannulation is the next preferred opinion when percutaneous
venous access fails
E. Intraosseous cannulation should be first choice for access -ANSWER- C. Blood
transfusion can be delievered through an intraosseous access

Regarding shock in the child, which of the following is FALSE?


A. Vital signs are age-related
B. Children have greater physiologic reserves than do adults
C. Tachycardia is the primary physiologic response to hypervolemia
D. The absolute volume of blood loss required to produce shock is the same as in
adults
E. An initial fluid bolus for resuscitation should approximate 20ml/kg of Ringers
lactate -ANSWER- D. The absolute volume of blood loss required to produce
shock is the same as in adults

A young man sustain a gunshot wound to the abdomen and is brought promptly to
the ED by prehospital personnel. His skin is cool and diaphoretic, and he is
confused. His pulse is thready and his femoral pulse is only weakly palpable. The
definitive treatment in managing this patient is to
A. Administer 0.neg blood
B. Apply external warming devices
C. Control internal hemorraghe operatively
D. Apply pneumatic anti shock garment
E. Infuse large volumes of IV crystalloid solution -ANSWER- C. Control internal
hemorraghe operatively
A four-year-old girl, weighing approximately 20kg is admitted in shock after an
automobile crash. The initialt fluid challenge or bolus should consist of Ringers
lactate solution in the volume of
A. 200ml
B. 400ml
C. 440ml
D. 600ml
E. 880ml -ANSWER- B. 400ml (20ml/kg)

All of the following are considered minimal precautions for the prevention of the
spread of communicable diseases during resuscitation, EXCEPT:
A. goggles
B. Face mask
C. Water-impervious gown
D. Water-impervious leggings
E. Needle-impenetrable sterile gloves -ANSWER- E. Needle-impenetrable sterile
gloves

30-year-old woman fell down four stair landing on concrete. Unconscious for 5
minutes after the fall, full consciousness during 10 minute transport to hospital,
GCS 15, complaint is a slight headache, 30 minute later she is unresponsive with
GCS 6 and left pupil is large. -ANSWER- Epidural hematoma

A young male fallen from height with obvious flail chest. ABG shows pH 7,47.
What is the cause of this abnormality? -ANSWER- Pulmonary contusion

Cushings triad which occurs in cases of increased intracranial pressure? -


ANSWER- Bradycardia with irregular respirations and isolated increase in
SYSTOLIC BLOOD PRESSURE

In comparison with young adults, elderly patients exhibit which of the following
regarding brain injuries?
A. Increased cerebral blood flow
B. Less stretching of the bridging veins
C. Less subdural hematomas
D. Less brain contusions
E. Less mobility with angular acceleration and deceleration -ANSWER- D. Less
brain contusions
Which of the following will be missed by DPL? -ANSWER- Subcapsular
hematoma of the spleen (becauase it is a retroperitoneal organ)

Burn victim, core temperature is 34C. Whats next?


A. Escharotomy
B. Rewarm
C. Oxygen mask
D...
E.. -ANSWER- C. Oxygen mask

Which of the following is NORMAL in pregnancy?


A. increased residual lung volume
B. Decreased plasma volume
C. Decreased total RBC mass
D. Widened symphysis pubis
E. -ANSWER- D. Widened symphysis pubis

A 34-year-old man is brought to the ED after being pinned to the wall of building
by a cement truck. He is in obvious shock, and has deformities and marked
swelling of both thighs. Although no open wound are present, his shock:
A. Cannot be explained without concomitant pelvic fracture
B. Signifies a loss of approximately 15%
C. Is consistent with blood loss from bilateral femoral fracture
D. Will likely be reversed if appropriate traction splint are applied
E. Cannot be explained by his observed injuries unless a major arterial injury exist
-ANSWER- C. Is consistent with blood loss from bilateral femoral fracture

Prior to passage of urinary catheter in a man, it is essential to:


A. Examine the abdomen
B. Determine pelvic stability
C. Examine the rectum and perineum
D. Perform a retrograde urethrogram
E. Know the history and mechanism of injury -ANSWER- C. Examine the rectum
and perineum

The best guide for adequate fluid resuscitation of the burn patient is:
A. Adequate urinary output
B. Reversal of systemic acidosis
C. Normalization of the heart rate
D. A normal central venous pressure
E. 4ml/kg/percent body burn/24 hours -ANSWER- A. Adequate urinary output

A 36-year-old woman is beaten about the head and face and is brought to the local
community hospital in full spinal immobilization. Her BP is 13088, HR
70/minutes, and RR 18/minute. Pulse oximetry indicated 98% while she was given
100% O2 via a non rebreather mask. Her airway is clear. She has marked swellings
on her face and several lacerations of her scalp that are not actively bleeding. She
does not respond to verbal stimuli, but localizes to painful stiumuli and opens her
eyes. She moves all extremities equally. The remainder of her physical exam is
normal. There is no neurosurgeon at the local hospital. After ensuring the patient
airway, the most appropriate course of action is to:
A. Admit the patient to the hospital for observation
B. Obtain x-ray of her facial bones prior to transfer
C. Obtain complete x-ray evaluation of the cervical spine
D. Transfer the patient to a neurosurgeon without performing a CT-scan
E. Perform DPL or request abdominal ultrasonography -ANSWER- D. Transfer
the patient to a neurosurgeon without performing a CT-scan

For the trauma patient with cerebral edema, hypercarbia should be avoided to
prevent:
A. metabolic acidosis
B. Respiratory acidosis
C. Cerebral vasodilatation
D. Neurogenic pulmonary edema
E. Reciprocal high level of PaCO2 -ANSWER- C. Cerebral vasodilatation

A 29 y/o male is brought to the ED after being involved in a motor vehicular


collision when his car struck a bridge abutment. He is intoxicated, has GCS 13 and
complains of abdominal pain. His BP was 80mmHg systolic by palpation on
admission, but rapidly increased to 110/70 with the administration of IV fluid. His
heart rate is 120/minute. The chest x-ray show loss of aortic know, widening of
mediastinum, no rib fracture and no hemopneumothorax. Contrast angiography:
A. Is not indicated
B. Should be performed after CT scan of the chest
C. Is positive ofr aortic rupture in 80% of similar cases
D. Is not necessary if the CT-scan of the chest is normal
E. Should be performed after DPL -ANSWER- D. Is not necessary if the CT-scan
of the chest is normal

Important screening x-rays to obtain in the multiple system trauma patient are:
A. Skull, chest and abdomen
B. Chest, abdomen and pelvis
C. Skull cervical spine and pelvis
D. Cervical spine, chest and pelvis
E. Cervical spine, chest and abdomen -ANSWER- D. Cervical spine, chest and
pelvis

All of the following statement regarding pulse oxymetry are true EXCEPT
A. excessive surrounding room light can interfere with the accuracy of the reading
B. Significant levels of dysfunctional hemoglobin can affect the accuracy of the
reading
C. It provides a continuous measurement of the partial pressure of oxygen
D. It is dependent on differential light absorption by oxygenated and deoxygenated
hemoglobin
E. It provides a continuous, non-invasiv measurement of pulse rate that is updated
with each HR -ANSWER- C. It provides a continuous measurement of the partial
pressure of oxygen

Bronchial intubation at the right or left mainstem bronchus can easily occuring
during infant endotracheal intubation because
A. The trachea is relatively short
B. The distance from the lips to the larynx is relatively short
C. The use of tubes without cuffs allow the tube to slip easily
D. The mainstem bronchi are less angulated in their relation to the trachea
E. Do litte friction exist between endotracheal tube and the wall of the trachea. -
ANSWER- A. The trachea is relatively short

A 52 y/o woman sustaining 50% total body surface burns in an explosion. She has
burns around the chest and both upper arms. Adequate resuscitation is initiated.
She is nasotracheally intubated and is being mechanically ventilated. Her
CarboxyHb level is 10%. Her arterial blood gas reveals PaO2 of 40mmHg, PaCo2
of 60mmHg and pH of 7,25. Appropriate immediate management at the time is to
A. Ensure adequate tissue perfusion
B. Increase the rate of fluid resuscitation
C. Add PEEP
D. Reassess for the presence of pneumothorax
E. Administer IV narcotics in small amounts -ANSWER- ?A. Ensure adequate
tissue perfusion

All of the following suggest urethral injury EXCEPT


A. scrotal hematoma
B. Blood in rectal lumen
C. Blod in external urethral meatus
D. High riding prostate on rectal exam
E. Absence of a palpable prostate on rectal exam -ANSWER- E. Absence of a
palpable prostate on rectal exam

Which one of the following is recommended method for threating frostbite?


A. Moist heat
B. Early amputation
C. Padding and elevation
D. Vasodilators and heparin
E. Topical application of silversulphadiazine -ANSWER- A. Moist heat

A 32-year-old mans right leg is trapped beneath his overturned car for nearly two
hours before he is extricated. On arrival in the ED, both lower extremities are cool,
mottled, insensate and motionless. Despite normal vital signs, pulses cannot be
palpated below the femoral vessels and the muscles of the lower extremities are
firm and hard. During the initial management of this patient, which of the followin
is most likely to improve chances for limb salvage?
A. Apply skeletal traction
B. Administering anticoagulant drugs
C. Administering trombolytic thearpy
D. Performing lower extremity fasciotomies
E. Immediately transfer the patient to a trauma care -ANSWER- D. Performing
lower extremity fasciotomies

A 26 y/o seat belted driver is brought to the ED after a car crash. Primary survey
reveals no evidence of serious injury except for diffuse, mild abdominal
tenderness. Bowel sounds are hypoactive and liver dullness is questionable.
Abdominal films reveal free air. The patient should
A. Undergo peritoneal lavage
B. Undergo promp celiotomy
C. Have a contrast x-ray of her GI-tract
D. Be carefully observed for further evidence of intraabdominal injury
E. Be suspected of having a ruptured diaphragm and accompanying pneumothorax.
-ANSWER- B. Undergo promp celiotomy

The least likely cause of a depressed level of consiousness in the multisystem


injured patient is:
A. shock
B. Head injury
C. Hyperglycemia
D. Impaired oxygenation
E. Alcohol and other drugs -ANSWER- C. Hyperglycemia

Which one of the following statements concerning massive hemothorax is true?


A. It is usually caused by blunt thoracic trauma
B. It is commonly confused with pneumothorax
C. The diagnosis should be confirmed by upright, plain chest x-ray prior to
treatment
D. The initial draining of 1000ml of blood after chest tube insertion requires
immediate thoracotomy
E. The condition should be suspected in situations with shock and unilateral absent
breath sounds -ANSWER- E. The condition should be suspected in situations with
shock and unilateral absent breath sounds

During primary and secondary survey, the patient injured by blunt trauma should
be completely immobilized until
A. The neurologic exam has been completed
B. The patient is transferred to definitive care
C. The patient is able to indicate that he has no neck pain
D. A spinal fracture has been excluded by x-ray
E. The patient complains of potential pressure sores due to the spine board. -
ANSWER- D. A spinal fracture has been excluded by x-ray

Cervical spine injury


A. Is excluded by a normal neurologic exam
B. Is not present if the patient has normal range of motion
C. Can be detected safely by careful flexion and extension of the neck
D. Can be excluded by a crosstable lateral x-ray of the c-spine
E. May be first manifested by neurologic deficit after movement of the neck -
ANSWER- E. May be first manifested by neurologic deficit after movement of the
neck

An 18 y/o man is brought to the ED after smashing his motorcycle into a tree. He
is conscious and alert, but paralyzed in both legs and arms. His skin is pale and
cold. He complains of thirst and difficulty in breathing. His airway is clear. His BP
is 60/40 and his pulse is 140 bpm. Breath sounds are full and equal bilaterally. He
should
A. Undergo exploratory celiotomy
B. Be treated for neurogenic shock
C. Be treated for hypovolemic shock
D. Undergo immediate nasotracheal intubation
E. be placed in cervical traction tongs before any other treatment is instituted -
ANSWER- C. Be treated for hypovolemic shock

The principle of balanced resuscitation is:


A) Permissive hypotension and early plasma infusion
B) Equal amounts of crystalloid and colloids
C) Simultaneous management of breathing and circulation
D) Maintenance of a normal acid base balance
E) Achieving a pulse rate <90 -ANSWER- A) Permissive hypotension and early
plasma infusion

Healthy young male in a motor vehicle crash is brought to the emergency


department with a blood pressure of 84/60, pulse 123, GCS 10. The patient moans
when his pelvis is palpated. After initiating fluid resuscitation, the next step in
management is:
A) Placement of a pelvic binder
B) Transfer to a trauma center
C) Pelvic x-ray
D) Insert urinary catheter
E) Repeat examination of pelvis -ANSWER- A) Placement of a pelvic binder

22 year old male fall from 2m, large right pneumothorax. Chest tube placed,
connected to drainage. Control x-ray shows pneumothorax, third x-ray reveals
pneumothorax -ANSWER- Tracheobronchal injury

Which one of the following is not a feature of neurogenic shock?


A. Increased venous capaitance
B. Decreased systemic vascular resistance
C. Increased cardiac output
D. Warm skin -ANSWER- C. Increased cardiac output

Tension pneumothorax can be caused by:


A. Flail chest
B. Cardiac tamponade
C. Clamping of a chest tube
D. All of the above -ANSWER-
Which one of the following brain stem reflexes is not assessed when assessing for
brain death?
A. Dolls eyes
B. Oculovestibular reflex
C. Gag reflex
D. Argyll Robertson pupil -ANSWER-

What is the simplest way to open the airway in an unconscious patient?


A. Pull out the tongue
B. Tilt head and lift chin
C. Lift neck from behind
D. Jaw thrust -ANSWER-

What is the approximate time the brain can be anoxic before developing
irreversible damage?
A. 10 min
B. 5 min
C. 2 min
D. 20 min -ANSWER-

You are treating a truama patient and attempt a definitive airway by intubation.
However, the vocal cords are not visible. What tool would be the most valuable for
achieving successful intubation?
A. Gum elastic bougie
B. Lateral cervical spine x-ray
C. Nasopharyngeal airway
D. Oxygen
E. Laryngeal mask airway -ANSWER- A. Gum elastic bougie

A 79 year old female is involved in a motor vehicle crash and presents to the ED.
She is on Coumadin and a beta blocker. Which of the following statements is true
concerning her management?
A. The risk of subdural hemorrhage is decreased
B. Absence of tachycardia indicates that the patient is hemodynamically normal
C. Non-operative management of abdominal injuries is more likely to be
successful in older adults than in younger patients
D. Vigorous fluid resuscitation may be associated with cardiorespiratory failure
D. epinephrine should be infused immediately for hypotension -ANSWER- D.
Vigorous fluid resuscitation may be associated with cardiorespiratory failure
The most common acid base disturbance encountered in injured pediatric patients
is caused by:
A. Hemorrhage
B. Changes in ventilation
C. Renal failure
D. Injudicious bicarbonate administration
E. Insufficient sodium chloride administration -ANSWER- B. Changes in
ventilation?

A 17 year old female is brought to the ED following a 2 meter fall onto concrete.
She is unresponsive and found to have a RR of 32, BP 90/60 and HR 68. The first
step in treatment is:
A. Adminstering vasopressors
B. Establishing IV access for drug-assisted intubation
C. Seeking the cause of her decreased level of consciousness
D. Applying oxygen and maintaining airway
E. Excluding hemorrhage as a cause of shock -ANSWER- D. Applying oxygen
and maintaining airway

A 25 year old male is brought to the ED following a bar fight. He has altered
consciousness, open his eyes on command, moans without forming discernible
words, and localizes to painful stimuli. Which one of the following statements
concerning this patient is true?
A. Hyperoxia should be avoided
B. CT scanning is an important part of neurological assessment
C. Mandatory intubation to protect his airway is required
D. His GCS suggest severe head injury
E. His level of consciousness can be solely attributed to elevated blood alcohol. -
ANSWER- B. CT scanning is an important part of neurological assessment

Han har GCS på 10-11, således passer det ikke med svaralternativ D som tilsvarer
GCS <9. Ettersom GCS er >8 passer heller ikke svaralternativ C. E er feil da det
ikke kan gi så lav GCS

Which one of the following statements regarding genitourinary injuries is true?


A. Urethral injuries are associated with pelvic fractures
B. All patients with microscopic hematuria require evaluation of genitourinary
tract
C. Patient presenting with gross hematuria and shock will have a major renal injury
as the source of hemorrhage
D. Intraperitoneal bladder injuries are usually managed definitively with a urinary
catether
E. Urinary catheters should be placed in all patients with pelvic fractures during
the primary survey -ANSWER- A. Urethral injuries are associated with pelvic
fractures
B er feil - det er pasienter med makroskopisk hematuri og/eller pas. med
mikroskopisk hematuri og sjokk som det er aktuelt å gjøre CT av. C kan være
riktig, men kan ikke si det sikkert at det er nyrene som er blødningskilden. D er feil
- ekstraperitoneal blæreskade behandles med kateter, og E er feil da man først må
undersøke for uretraskade

Cardiac tamponade:
A. Requires surgical intervention
B. Is defintively managed by needle pericardiocentesis
C. Is easily diagnosed by discovery of Becks triad in the ED
D. Is indicated by Kussmaul breathing
E. Is most common with blunt thoracic trauma and anterior rib fractures -
ANSWER- A. Requires surgical intervention

A 6 month old infant, being held in her mothers arms, is ejected on impact from a
vehicle that is struck head on by an oncoming car traveling at 65kph. The infant
arrives in the ED with multiple facial injuries, is lethargic, and is in severe
respiratoy distress. Respiratory support is not effective using a bag mask device,
and her oxygen saturation is falling. Repeated attempts at orotracheal intubation
are unsuccessful. the most appropriate procedure to perform next is:
A. perform needle cricothyroidotomy with jet insufflation
B. Administer heliox and racemic epinephrine
C. Perform nasotracheal intubation
D. Perform surgical cricothyroidotomy
E. Repeat orotracheal intubation -ANSWER- A. perform needle cricothyroidotomy
with jet insufflation

Which one of the following injuries is adressed in the secondary survey?


A. Bilateral femur fractures with obvious deformity
B. Open fracture with bleeding
C. Mid thigh amputation
D. Unstable pelvic fracture
E. Forearm fracture -ANSWER- E. Forearm fracture
A 22 year old male present following a motorcycle crash. He complains of the
inability to move his legs. His BP is 80/50, HR 70, RR 18 and GCS 15. Oxygen
saturation is 99% on 21 nasal prongs. Chest x-ray, pelvic x-ray and FAST are
normal. Extremities are normal. His management should be:
A: 2L of iv . crystalloid and two units of pRBCs
B. 2L of iv crystalloid and vasopressors if BP does not respond
C. 2L of iv. crystalloid, mannitol and iv steroids
D. Vasopressors and laparotomy
E. 1 unit of albumin and compression stockings -ANSWER- B. 2L of iv crystalloid
and vasopressors if BP does not respond

A 35 year old female sustains multiple injuries in a motor vehicle crash and is
transported to a small hospital in full spinal protection. She has a GCS of 4 and is
being mechanically ventilaed. I.v access is established and warmed crystalloid is
infused. She remains hemodynamically normal and full spinal protection is
maintained. Preparations are made to transfer her to another facility for definitive
neurosurgical care. Prior to transport, which of the following tents or treatments is
mandatory?
A. FAST exam
B. Lateral cervical spine xray
C. Chest x-ray
D. Administration of methylprednisolon
E. CT of abdomen -ANSWER- C. Chest x-ray

A 22 year old male is assaulted in a bar. A semi-rigid cervical collar is applied and
he is immobilized on a spine board. On initial examination, his vital signs are
normal, and his GCS is 15. Which of the following is an indication for CT in this
patient with possible minor traumatic brain injury?
A. Blood alcohol concentration of 0,16%
B. Presence of an isolated 10cm scalp laceration
C. Presence of a mandibular fracture
D. Presence of hemotympanum
E. History of assault -ANSWER- D. Presence of hemotympanum

Which one of the following statement is true?


A. Hypotonic fluids should be used to limit brain edema in patients with severe
head injury
B. Elevated intracranial pressure will not affect cerebral perfusion
C. CSF cannot be displaced from cranial vault
D. Cerebral blood flow is increased when PaCO2 is below 30mmHg --> below
50mmHg
E. Autoregulation of cerebral blood flow normally occurs between cerebral
perfusion pressure of 50 to 150mmHg -ANSWER- D. Cerebral blood flow is
increased when PaCO2 is below 30mmHg --> below 50mmHg

A 40 year old obese patient with GCS of 8 requries a CT scan. Before transfer to
the scanner, you should:
A. give more sedative drugs
B. Insert a defintive airway
C. Insert a multilumen esophageal airway
D. Request a lateral cervical spine film
E. insert a nasogastric tube -ANSWER- B. Insert a defintive airway

A 30 year old male is brought toe the hospital after falling 6 meters. Inspection
reveals an obvious flail chest on the right. The patient is tachypneic. Breath sounds
are present and symmetrical. There is no significant hyperresonance or dullness.
Arterial blood gas obtained while the patient recieves oxygen by face mask are:
PaO2 of 45mmHg (6Kpa), PaCO2 of 28mmHg (3,7 Kpa) and pH of 7,47. The
component of injury that most likely responsible for abnormalities in this patients
blood gas is:
A. Hypoventilation
B. Pulmonary contusion
C. Hypovolemia
D. Small pneumothorax
E. Flail chest -ANSWER- B. Pulmonary contusion

A 14 year old female is brought to the ED after falling from a horse. She is
immobilized on a long spine board with a hard collar and blocks. Cervical spine x-
ray:
A. Will show cervical spine injury in more than 20% of these patients
B. Will exclude cervical spine injury if no abnormalities are found on the x-rays
C. Are not needed if she is awake, alert, neurologically normal, and has no neck
pain or midline tenderness
D. Should be performed before adressing potential breating or circulatory problems
E. May show atlanto-occipital dislocation if the Powers ratio is <1 --> 1:noraml,
>1 anterior, <1 posterior -ANSWER- C. Are not needed if she is awake, alert,
neurologically normal, and has no neck pain or midline tenderness

The most specific test to evaluate for injuries of solid abdominal organ is:
A. Abdominal x-ray
B. Abdominal ultrasound
C. DPL
D. Frequent abdominal examination
C. CT of abdomen and pelvis -ANSWER- C. CT of abdomen and pelvis

The most important consequence of inadequate organ perfusion is:


A. Vasodilatation
B. Multiple organ failure
C. Decreased base deficit
D. Acute glomerulonephritis
E. Increased cellular ATP production -ANSWER- B. Multiple organ failure

A 23 year old construction worker is brought toe the ED after falling more than 9
meters from scaffolding. He is complaining bitterly of lower abdominal and lower
limb pain, and has obvious deformity of both lower legs with bilateral open tibial
fractures. Which one of the following statement concerning this patient is true?
A. Pelvic injury can be ruled out based on the mechanism of injury
B. Blood loss from the lower limb is most likely cause of his hypotension
C. Spinal cord injury is the most likely cause of his hypotension
D. X-ray of the chest and pelvis are important adjuncts in his assessment
E. Aortic injury is the most likely cause of his tachycardia. -ANSWER- D. X-ray
of the chest and pelvis are important adjuncts in his assessment

A 82 year old male falls down five stairs and presents to the ED. All following
statements are true statements regarding his condition compared to a younger
patient with similar mechanism, except:
A. He is more likely to have had a contracted circulatory volume prior to his injury
B. His risk of cervical spine injury is increased due to degeneration, stenosis and
loss of disk compressibility
C. His risk of occult fractures is increased
D. His risk of bleeding may be increased
E. Intracranial hemorrhage will become symptomatic more quickly -ANSWER- E.
Intracranial hemorrhage will become symptomatic more quickly

A 25 year old female in the third trimester of pregnancy is brought to the ED


following a high speed motor vehicle crash. She is conscious and immobilized on a
long spine board. Her RR is 24 bpm, HR is 120bpm, and BP is 70/50mmHg. The
lab results show a PaCO2 of 40mmHg. Which one of the following statements
concerning this patient is true?
A. Fetal assessment should take priority
B. Log-rolling the patient to the right will decompress the vena cava
C. Rh-immunoglobulin therapy should be immediately administered
D. Vasopressors should be given to the patient
E. The patient has likely impending respiratory failure. -ANSWER- E. The patient
has likely impending respiratory failure.
Minute ventilation increases primarily due to an increase in tidal volume.
Hypocapnia (PaCO2 of 30 mm Hg) is therefore common in late pregnancy. A
PaCO2 of 35 to 40 mm Hg may indicate impending respiratory failure during
pregnancy.

Lateral cervical spine film:


A. Must be performed in the primary survey
B. Can exclude any significant spinal injury
C. Are indicated in all trauma patients
D. Should be combined with clinical exam, AP and odontoid, or CT
E. Require the following films: oblique views, AP, odontoid and flexion extention
views prior to spinal clearance in trauma patients -ANSWER- D. Should be
combined with clinical exam, AP and odontoid, or CT

A 30 year old male is stabbed in the right chest. On arrival in the ED, he is very
short of breath. His HR is 120bpm, BP is 80/50. His neck veins are flat. On
auscultation of the chest, there is diminshed air entry on the right side, and there is
dullness posteriorly on percussion. These findings are most consistent with:
A. Hemothorax
B. pericardial tamponade
C. Tension pneumothorax
D. Hypovolemia from the liver injury
E. Spinal cord injury -ANSWER- A. Hemothorax

A specific aspect of the treatment of thermal injury is:


A. Chemical burn require the immediate removal of clothing
B. Patients who sustain thermal injury are at lower risk for hypothermia
C. Patients with circumferential truncal burns need prompt fasciotomies
D. Electrical burn are associated with extensive skin necrosis (from entry point to
exit)
E. The parkland formula should be used to determine adequacy of resucitation. -
ANSWER- A. Chemical burn require the immediate removal of clothing
A 15 year old male is brought to the ED after being involved in a motor vehicle
crash. He is unconscious and was intubated at the scene by emergency personnel.
Upon arrival at the ED, the patients oxygen saturation is 92%, HR is 96 bpm and
BP is 150/85 Breath sounds are decreased on the left side of the thorax. The next
step is:
A. Immediate needle crycothyroidotomy
B. Reassess the position of the endotracheal tube
C. Chest tube insertion
D. Immediate needle thoracentesis
E. Obtain a chest x-ray -ANSWER- B. Reassess the position of the endotracheal
tube

The first priory in management of a long bone fracture is:


A. Reduction of pain
B. Prevention of infection in case of an open fracture
C. Prevention of further soft tissue injury
D. Improve long term function
E. Control of hemorrhage -ANSWER- E. Control of hemorrhage

A 30 year old female is brought to the ED after being injured in a motor vehicle
crash. Her initialt BP is 90/60mmHg, and her HR is 122bpm. She responds to rapid
infusion of 1L crystalloid solution with a rise in her BP to 118/88 and decrease in
her HR to 90bpm. Her pressure suddenly decreased to 96/66. The least likely cause
of her hemodynamic change is:
A. Ongoing blood loss
B. Blunt cardiac injury
C. Traumatic brain injury
D. Inadequate resuscitation.
E. Tension pneumothorax -ANSWER- C. Traumatic brain injury

Limb-threatening extremity injuries:


A. Require a torniquet
B. Should be defintively managed by application of a traction split.
C. Are rarely present without an open wound
D. Are characterized by the presence of ischemic or crushed tissue.
E. Indicate a different order or priorities for the patients initial assessment and
resuscitation. -ANSWER- B. Should be defintively managed by application of a
traction split.
A 29 year old female arrives in the ED after being involved in a motor vehicle
crash. She is 30 weeks pregnant. She was restrained with a lap and shoulder belt,
and an airbag deployed. Which one of the following statement best decribes the
risk of injury?
A. Deployment of the airbag increased the risk of fetal loss
B. The risk of premature fetal delivery and death is reduced by the use of restraints
C. The use of seatbelts is associated with increased risk of maternal death.
D. The mechanism of injury suggest the need for emergency ceasarean section due
to the risk of impending abruptio placentae
E. The deployment of the airbag increases the risk of maternal abdominal injury -
ANSWER- B. The risk of premature fetal delivery and death is reduced by the use
of restraints

Supraglottic airway devices:


A. Are equivalent to endotracheal intubation
B. Require neck extension for proper placement
C. Are preferable to endotracheal intubation in a patient who cannot lie flat
D. Are of value as part of a difficult or failed intubation plan
E. Provide one form of definitive airway -ANSWER- D. Are of value as part of a
difficult or failed intubation plan

A 25 year old male is brought to the hospital after sustaining partial and full
thickness burns involving 60% of his body surface area. His right arm and hand are
severely burned. There are obvious full thickness burns of the entire right hand and
a circumferential burn of the right arm. Pulses are absent at the right right wrist and
are not detected by doppler examination. The first step in management of the right
upper extremity should be:
A. Fasciotomy
B. Angiography
C. Escharatomy
D. Heparinization
E. Tangential excision -ANSWER- C. Escharatomy

Hypertension following a headinjury:


A. Should be treated to reduce intracranial pressure
B: Indicates pre-existing hypertension
C. May indicate imminent herniation from critically high intracranial pressure
D. Mandates prompt administration of mannitol
E. Should prompt burr hole drainage of potentialt subdural hematomas. -
ANSWER- C. May indicate imminent herniation from critically high intracranial
pressure

Initial treatment of frostbite injuries involves:


A. application of dry heat
B. Debridement of hemorrhagic blisters
C. Early amputation to prevent septic complications
D Rapid rewarming of the body part in circulation warm water
E. Massage of the affected area -ANSWER- D Rapid rewarming of the body part
in circulation warm water

Signs and symptoms of airway compromise include all of the following, EXCEPT:
A. change in voice
B. Stridor
C. Tachypnea
D. Dyspnea and agitation
E. Decreased pulse pressure -ANSWER- E. Decreased pulse pressure

Which one of the following statements is true regarding a pregnant patient who
presents following blunt trauma?
A. Early gastric decompression is important
B A Hb level of 10g/dl indicates recent blood loss
C. The CVP response to volume resuscitation is blunted in pregant patients
D. A lap belt is the best form of restraint due to the size of the gravid uterus
E. A PaCO2 of 40mmHg provides reassurance about the adequacy of respiratory
function -ANSWER- A. Early gastric decompression is important
Hb kan være lav av andre grunner, CVP-responen på væske er lik som hos ikke-
gravide, lap+shoulder belt er beste form for restraint, og hypocapnia er vanlig i
graviditeten slik at normalverdi kan indikere resp.svikt.

A 70 year old suffers blunt chest traume after bein struck by a car. On presentation,
his GCS is 15, BP is 145/90, HR is 72. RR is 24 and O2-sat on 5L is 91%. Chest x-
ray demonstrates multiple right sided rib fractures. ECG demonstrates normal sinus
rythm with no conduction abnormalities. Management should include:
A. Placement of a 22Fr right sided chest tube
B. Serial troponins and cardiac monitoring
C. Monitoried i.v analgesia
D. Thoracic splinting, taping and immobilization
E. Bronchoscopy to exclude tracheobronchial injury -ANSWER- C. Monitoried i.v
analgesia.
Ikke påvist pneumo eller hemothorax, derfor ikke thoraxdren. Troponiner gir ingen
tilleggsinformasjon utover hva EKG kan gi. Pasienter med stump skade mot hjertet
med diagnostisert med konduksjonsabnormalitet (unormalt EKG) er i riskosonen
for å få arrytmier og skal overvåkes i 24 timer med EKG. Pasienter UTEN EKG-
forandringer behøver ikke videre monitorering.

A 15 year old male present following a motorcycle crash. INitial examinations


reveals normal vital signs. There is a large bruise over his epigastrium that extends
to the left flank. He has no other apparent injuries. A CT-scan of the abdomen
demonstrate a ruptured spleen surrounded by a large hematoma and fluid in the
pelvis. The next step in the patients management is:
A. Splenic artery embolization
B. Pneumococcal vaccine
C. Urgent laparotomy
D. Surgical consult
E. Transfer to a pediatrician -ANSWER- D. Surgical consult
Hemodynamisk stabil, ingen fri luft.

A 30 year old male present with a stab wound to the abdomen BP is 85/60, HR
130, RR 26 and GCS 14. Neck veins are flat and chest examination is clear with
bilateral breath sounds. Optimal resusciation should include:
A. Transfusion of FFP and platelts
B. 500ml of hypertonic saline and transfusion of pRBCs
C. Resuscitation with crystalloid and pRBC until base excess is normal
D. Fluid resuscitation and angioembolization
E. Preparation for laparotomy while initiating fluid resuscitation -ANSWER- E.
Preparation for laparotomy while initiating fluid resuscitation

Initial resuscitation in adult patient should:


A. Be with 1-2L of crystalloid, monitorin the patients response
B. Use crystalloid to normalize BP
C. Use permissive hypotension in patients with head injury
D. Be with a non blood colloid solution
E. Be a minimum of 2L Crystalloid in all trauma patient prior to adminstering
blood -ANSWER- A. Be with 1-2L of crystalloid, monitoring the patients response

Compared with adults, children have:


A. A longer, wide, funnel shaped airway
B. A less pliable, calcified skeleton
C. Lower incidence of bony injury with neurogenic shock
D. A relatively smaller head and larger jaw
E. Anterior displacement of C5 and C6 -ANSWER- B. A less pliable, calcified
skeleton

A 30 year old male presents after a motor vehicle crash. Vital signs are RR 18, HR
88, BP 130/72, GCS 13. Laparotomy is indicated when
A. There is distinct seat belt sign over the abdomen
B. The CT-scan demonstrates a grade 4 hepatic injury
C. There is evidence of extraperitoneal bladder injury
D. CT demonstrates retroperitoneal air
E. The abdomen is distended with localized right upper quadrant tenderness. -
ANSWER- D. CT demonstrates retroperitoneal air

Indikasjoner for laparotomi:


Blunt abdominal trauma with hypotension, with a positive FAST or clinical
evidence of intraperitoneal bleeding, or without another source of bleeding
• Hypotension with an abdominal wound that penetrates the anterior fascia
• Gunshot wounds that traverse the peritoneal cavity
• Evisceration
• Bleeding from the stomach, rectum, or genitourinary tract following penetrating
trauma
• Peritonitis
• Free air, retroperitoneal air, or rupture of the hemidiaphragm
• Contrast-enhanced CT that demonstrates ruptured gastrointestinal tract,
intraperitoneal bladder injury, renal pedicle injury, or severe visceral parenchymal
injury after blunt or penetrating trauma

A 20 year old male is brought to the hospital approximately 30 minutes after being
stabbed in the chest. There is 3cm wound just medial to the left nipple. His BP is
70/33 and HR is 140. Neck and arm veins are distended. Breath sounds are normal.
Heart sounds are diminshed, iv access has been established and warm crystalloid is
infusing. The next most important aspect of immediate management is:
A. CT scan of the chest
B. 12 lead ECG
C. Left tube thoracostomy
D. Begin infusion of RBCs
E. FAST exam -ANSWER- E. FAST exam For å se etter tamponade? Normale
resp.lyder bilateralt taler mot trykkpneumothorax. EKG vil ikke gi noe
informasjon. Thoracostomy ikke indisert da det ikke er mistenkt
pneumo/hemothorax. CT uaktuelt pga hemodynamisk ustabil

A 47 year old house painter is brought to the hospital after falling 6 meters from a
ladder and landing straddled on a fence. Examination of his perineum reveals
extensive ecchymosis. There is blood in the external urethreal meatus. The initial
diagnostic study for evaluation of the urinary tract in this patient should be:
A. Cystoscopy
B. Cystography
C. IV pyelography
D. CT scan
E. Retrograde urethrography -ANSWER- E. Retrograde urethrography

Neurogenic shock has all of the following classic characteristics except which one:
A. Hypotension
B. Vasodilatation
C. Bradycardia
D. Neurologic deficit
E. Narrowed pulse pressure -ANSWER- E. Narrowed pulse pressure

Which one of the following should be performed first in any patient whose injuries
may include multiple closed extremity fractures?
A. A thorough assessment of four limb perfusion
B. Maneuvers to prevent necrosis of the skin
C. Extremity compartment syndrome release
D. Ensuring adequate oxygenation and ventilation
E. Evaluation for occult crush syndrome -ANSWER- D. Ensuring adequate
oxygenation and ventilation
Patients with a GSC of less than ___ usually require intubation. -ANSWER- 8

The "A" in ABCD stands for _______. -ANSWER- Airway maintenance with CERVICAL
SPINE PROTECTION

You should assume that any patient in a multisystem trauma with an altered level of
consciousness or blunt injury above the clavicle has what type of injury? -ANSWER-
Cervical spine injury

Flail chest is invariably accompanied by ______ which can interfere with blood
oxygenation. -ANSWER- pulmonary contusion - do NOT over fluid resuscitate these
patients!
Hypotension is caused by _____ until proven otherwise. -ANSWER- hypovolemia

When you don't have/can't get a blood pressure, what are three things to look for when
evaluating perfusion. -ANSWER- 1. Level of consciousness (brain perfusion), 2. Skin
color (ashen face/grey extremities) 3. Pulse (bilateral femoral - thready/tachy)

Elderly patients have a limited ability to ______ to compensate for blood loss. -
ANSWER- increase heart rate

Resuscitation fluids should be warmed 39 degrees Celsius (102.2 F). Can you use a
microwave to do this? -ANSWER- YES - for CRYSTALLOID ONLY (but NOT for blood
products).

Urinary catheters are good for assessing renal perfusion and volume status. List 5 signs
of urethral injury that might prevent you from inserting one. -ANSWER- Blood at urethral
meatus, perineal ecchymosis, blood in scrotum, high-riding/non-palpable prostate,
pelvic fracture

Which arm should you NOT put a pulse-ox on? -ANSWER- The arm with a blood
pressure cuff on it

Name two anatomical things that can interfere with doing a FAST scan. -ANSWER-
Obesity & intraluminal bowel gas

When should radiographs be obtained? -ANSWER- During the SECONDARY survey.

How do you get an ample patient history? -ANSWER- A=Allergies, M=Medications,


P=PMH/Pregnancy, L=Last meal, E=Events/Environment of injury

Why might you want a Bair Hugger for a patient who smells of alcohol? -ANSWER-
Vasodilation can lead to hypothermia

What things are you looking for when you do a DRE in a trauma? -ANSWER- Blood,
high-riding prostate (in males), and sphincter tone

What should you do for every female patient? -ANSWER- Pregnancy test (females of
childbearing age)

Adult patients should maintain UOP of at least ___ mL/kg/hr. Kids should have at least
___ mL/kg/hr. -ANSWER- Adults 0.5 mL/kg/hr, Kids 1.0 ml/kg/hr

Preventing hypercarbia is critical in patients who have sustained a _____ injury. -


ANSWER- head

What two places would you LOOK at a patient if you suspect hypoxemia? -ANSWER-
Lips and fingernail beds
Patients may be abusive and belligerent because of _____, so don't just assume it's
due to drugs, alcohol, or the fact that they are just inherently a jerk. -ANSWER- hypoxia

Can a patient breathe on their own after complete cervical cord transection? -ANSWER-
Yes, if the phrenic nerves (C3-C5) are spared. This will result in "abdominal" breathing.
The intercostal muscles will be paralyzed though.

Can you use an OPA (Guedel) in a conscious patient? -ANSWER- No, it could make
them vomit. An NPA (trumpet) would be okay.

Bougies are typically inserted blindly, how do you know you are in the trachea and not
the esophagus? -ANSWER- You can feel the "clicks" as the distal tip rubs against the
cartilaginous tracheal rings, or it will deviate right or left when entering either bronchus
(usually at 50 cm).

What do you NOT want to hear if you ascultate a patient after placement of an ET tube?
-ANSWER- Borborygmi - rumbling or gurgling noises suggesting esophageal insertion.

What is the RSI dose for etomidate? -ANSWER- 0.3 mg/kg (usually 20 mg)

What is the RSI dose for sux? -ANSWER- 1-2 mg/kg (usually 100 mg)

How does etomidate affect blood pressure? -ANSWER- It doesn't - at least it


SHOULDN'T have any significant effect on BP. Ketamine will increase BP, and propofol
and thiopental will both drop BP.

A RSI dose of sux usually lasts about ___ minutes. -ANSWER- 5

What hypnotic/sedative/induction agent do you NOT want to use for a severely burned
patient? -ANSWER- SUX - patients with severe burns, crush injuries, hyperkalemia, or
chronic paralytic/neuromuscular diseases should NOT get sux because of hyperkalemia
risk.

Oxygen should flow at 15L for needle cricothyroidotomy, and have a Y-connector for
insufflation if possible. What size needle do you use for adults? Kids? -ANSWER-
Adults 12-14 gauge, kids 16-18 gauge

Cricoid cartilage is the only circumferential support for the upper trachea in kids,
therefore surgical cricothyroidotomy is not recommended in kids under the age of ___. -
ANSWER- 12

In a "normal" patient without significant chest wall injury or lung disease, needle
cricothyroidotomy can provide adequate oxygenation for approximately ____ minutes. -
ANSWER- 30-45
For a patient with difficulty breathing, what things might you try before you provide a
surgical airway? -ANSWER- Chin-lift, jaw-thrust (NOT head-tilt while maintaining c-
spine precautions), OPA (guedel), NPA (trumpet), LMA, Combitube, ET tube +/- bougie

How do you know if an OPA/Guedel is the correct size for the patient? -ANSWER- A
correctly sized OPA will extend from the corner of the patient's mouth to the external
auditory canal.

What should do with the balloon on an ET tube/LMA/foley before you insert it? -
ANSWER- Inflate it to make sure it doesn't leak - then deflate and insert.

What size LMA do you use for kid, woman/small man, large woman/man? -ANSWER-
Kid: 3, Woman/small man: 4, Large woman/man: 5 (C3,4,5 keep the diaphragm alive)

The proper size ET tube for an infant is ______. -ANSWER- The same size as the
infant's nostril or little finger. (Usually size 3 for neonates, 3.5 for infants)

What size cuffed endotracheal tube do you use for an emergency cricothyroidotomy? -
ANSWER- 5 or 6

Use size 3 ET tubes for neonates, 3.5 for infants 0-6 months, and size 4 for infants 6-12
months. How do you calculate what size ET tube to use for toddlers and kids? -
ANSWER- Age/4 + 4 mm = internal diameter

Shock is defined as an abnormality of the circulatory system that results in inadequate


organ perfusion and tissue oxygenation. What are the 4 different types? -ANSWER-
Neurogenic, cardiogenic, hypovolemic, septic

The most common cause of shock in the injured trauma patient is _____. -ANSWER-
hemorrhage

Approximately ___% of the body's total blood volume is located in the venous circuit. -
ANSWER- 70

Why does shock actually reduce the total volume of circulating blood? -ANSWER-
Anaerobic metabolism --> can't make more ATP --> Endoplasmic then mitochondrial
damage --> lysosomes rupture --> sodium and WATER enter the cell, which SWELLS
and dies.

Which vasopressors should you use to treat hemorrhagic shock? What are the drug
doses? -ANSWER- NEVER use pressors for hypovolemic shock - use VOLUME
replacement. Pressors will worsen tissue perfusion in hemorrhagic shock.

Compensatory mechanisms may preclude a measurable fall in systolic blood pressure


until up to __% of the patient's blood volume is lost. -ANSWER- 30
Any patient who is cool and is tachycardic is considered to be ______ until proven
otherwise. -ANSWER- in shock

The definition of tachycardia depends on the patient's age. What heart rate is
considered tachycardic for infants, toddlers/preschoolers, school age/prebuscent, and
adults? -ANSWER- Infants >160, toddlers/preschoolers >140, school age/prebuscent
>120, adults >100

Elderly patients may not exhibit tachycardia in response to hypovolemia because of


limited cardiac response to catecholamines. Why else might not they get tachy? -
ANSWER- They might be on a beta-blocker or have a pacemaker.

A FAST scan is an excellent way to diagnose cardiac tamponade. What signs suggest
tamponade? -ANSWER- Becks's Triad: JVD, muffled heart sounds, and hypotension
(will be resistant to fluid therapy). Will also likely be tachycardic.

Patients with a tension pneumo and patients with cardiac tamponade may present with
many of the same signs. What findings will you see with a tension pneumo that you will
NOT see with tamponade? -ANSWER- Absent breath sounds and hyperresonance to
percussion over the affected hemithorax.

Immediate thoracic decompression is warranted for anyone with absent breath sounds,
hyperresonance to percussion, tracheal deviation, ____, and ____. -ANSWER- Acute
respiratory distress & subcutaneous emphysema

Can isolated intracranial injuries cause neurogenic shock? -ANSWER- NO

How do you calculate total blood volume in an adult? -ANSWER- 70 mL per kg body
weight. A 70 kg person has about 5 liters of circulating blood. (70*70=4900)

How do you calculate total blood volume in an child? -ANSWER- Body weight in kg x
80-90 mL

The blood volume of an obese person is calculated based upon their ______ weight. -
ANSWER- ideal

Fluid replacement should be guided by ________, not simply by the initial classification
(Class I-IV). -ANSWER- The patient's response to initial replacment

How much blood volume is lost with Class I Hemorrhage? -ANSWER- Up to 15%
Donating 1 pint, or ~500 mL of blood is about a 10% volume loss and would qualify as
Class I Hemorrhage!

How do you treat a Class I Hemorrhage? -ANSWER- You don't (usually). Transcapillary
refill and other compensatory mechanisms usually restore blood volume within 24
hours.
How much blood volume is lost with Class II Hemorrhage? -ANSWER- 15-30% (750-
1500 mL in a 70 kg adult)

How do you treat a Class II Hemorrhage? -ANSWER- Usually just crystalloid


resuscitation

Subtle CNS changes such as anxiety, fright, and hostility would be expected in a patient
with a Class __ Hemorrhage. -ANSWER- II

How much blood volume is lost with Class III Hemorrhage? -ANSWER- 30-40% (2000
mL in a 70 kg adult)

A patient with inadequate perfusion, marked tachycardia and tachypnea, significant


mental status change, and a measurable fall in systolic blood pressure likely has a
Class ___ Hemorrhage. -ANSWER- III or IV - These patients almost always require a
blood transfusion, which depends on their response to initial fluid resuscitation. The first
priority is stopping the hemorrhage.

Loss of more than ___% of blood volume results in loss of consciousness. -ANSWER-
50

How much blood volume is lost with Class IV Hemorrhage? -ANSWER- More than 40%.
Unless very aggressive measures are taken the patient will die within minutes.

A Class ___ Hemorrhage represents the smallest volume of blood loss that is
consistently associated wiht a drop in systolic blood pressure. -ANSWER- III

Up to ______ mL of blood loss is commonly associated with femur fractures. -


ANSWER- 1500

Unexplained hypotension or cardiac dysrhythmias (usually bradycardia from excessive


vagal stimulation) are often caused by ______, especially in children. -ANSWER-
gastric distention

How much crystalloid should you give an adult for an initial fluid resuscitation bolus? For
kids? -ANSWER- Adults: 2 liters, Kids: 20 mL/kg (may repeat and give as much as 60
mL/kg but with high reserve in kids, if they're in shock they should get blood sooner
rather than later.

Each mL of blood loss whould be replaced with ___ mL of crystalloid, thus allowing for
replacement of plasma volume lost into interstitial and intracellular spaces. -ANSWER-
3

Blood on the floor x four more is a mnemonic for occult blood loss where? -ANSWER-
Chest, pelvis, retroperitoneum, and thigh
For children UNDER 1 year of age, UOP should be ___ mL/kg/hr. -ANSWER- 2

Would patients in EARLY hypovolemic shock be acidodic or alkalotic? -ANSWER-


Alkalotic - respiratory alkalosis from tachypnea....followed later by mild metabolic
acidosis in the early phase of shock.

"Rapid responders" whose vital signs return to normal (and stay there) after fluid
resuscitation likely have/had a Class ___ Hemorrhage. -ANSWER- I or II

"Transient responders" are associated with Class ___ Hemorrhage. -ANSWER- II or III

What differential diagnoses should you always consider for "non-responders" following
fluid resuscitation? -ANSWER- NON-HEMORRHAGIC causes, e.g. tension
pneumothorax, tamponade, blunt cardiac injury, MI, acute gastric distention, neurogenic
shock...

Most patients receiving blood transfusions ____ need calcium replacement. -ANSWER-
don't

How should you position the patient before placing a subclavian or IJ line? -ANSWER-
SUPINE, head down 15 degrees to distend neck veins and prevent embolism, only turn
head away if C-SPINE HAS BEEN CLEARED FIRST.

How long can you keep an IO line in? -ANSWER- Intraosseous infusion should be
limited to emergency resuscitation and shoudl be discontinued as soon as other venous
access is obtained.

Where do you want to make an incision for a saphenous vein cutdown and how long
should your incision be? -ANSWER- The saphenous vein can be accessed
approximately 1 cm anterior and 1 cm superior to the medial malleolus. Make a 2.5 cm
transverse incision through the skin and SQ tissue, careful not to injure the vessel.

A patient arrives to the trauma bay intubated and there are absent breath sounds over
the left hemithorax, where should you place your decompression needle? -ANSWER-
This may NOT be a pneumothorax, for intubated patients always suspect a right main-
stem before attempting needle decompression.

Where would you insert a large caliber needle to decompress a tension pnuemothorax?
-ANSWER- Into the 2nd intercostal space in the midclavicular line of the affected
hemithorax.

For an open pneumothorax, (sucking chest wound) air passes preferentially through the
chest wall defect (least resistance) if the diameter of the defect is at least ___ the
diameter of the trachea. -ANSWER- 2/3
Flail chest results from multiple rib fractures - by definition this would be ___ or more
ribs, fractured in ___ or more places. -ANSWER- 2 or more ribs fractured in 2 or more
places

Both tension pneumothorax and massive hemothorax are associated with decreased
breath sounds on auscultation, so you can tell which it is by _______. -ANSWER-
Percussion - hyperresonant with pnuemo, dull with hemothorax.

If a patient doesn't have JVD, does this mean they don't have a tension pneumo or
tamponade? -ANSWER- No, they might have a massive internal hemorrhage and be
hypovolemic.

By definition, how much blood is in the chest cavity to call it a "massive hemothorax"? -
ANSWER- 1500 mL or 1/3 or more of the patient's total blood volume. (Some also
define it as continued blood loss of 200 mL/hr for 2-4 hours- but ATLS does NOT use
this rate for any mandatory treatment decisions).

What size chest tube might you use to evacuate a massive hemothorax? -ANSWER-
#38 French - inserted at the 4th or 5th intercostal space, just anterior to the midaxillary
line.

What is Kussmaul's sign? -ANSWER- A rise in venous pressure with inspiration while
breathing spontaneously, and is a true paradoxical venous pressure abnormality
associated with cardiac tamponade.

How well do CPR compressions work on someone with a penetrating chest injury and
hypovolemia? -ANSWER- "Closed heart massage for cardiac arrest or PEA is
INEFFECTIVE in patients with hypovolemia." Patients with PENETRATING thoracic
injuries who arrive pulseless, but with myocardial electrial activity, may be candidates
for an ED thoacotomy.

Are patients with PEA who have sustained blunt thoracic injuries candidates for an ED
thoracotomy? -ANSWER- NO - Only PEA with PENETRATING thoracic injuries should
get an ED thoracotomy.

An ED thoracotomy can allow you to do what? -ANSWER- Evacuate pericardial blood,


direcly control hemorrhage, cardiac massage, cross-clamp the descending aorta to slow
blood loss below the diaphragm and increase perfusion to the heart and brain.

For a patient with a traumatic simple pneumothorax, what should you do BEFORE you
start positive pressure ventilation or take them to surgery for a GA? -ANSWER- CHEST
TUBE - positive pressure ventilation can turn a simple pneumo into a tension pneumo,
so put in a chest tube first.
Should you evacuate a simple hemothorax if it is not causing any respiratory problems?
-ANSWER- YES - A simple hemothorax, if not fully evacuated, may result in a retained,
clotted hemothroax with lung entrapment or, if infected, develop into an empyema.

A pneumothorax associated with a persistent large air leak after tube thoracostomy
suggests a _______ injury. -ANSWER- tracheobronchial - Use bronchoscopy to
confirm, you may need more than one chest tube before definitive operative
management.

What radiographic findings are suggestive of traumatic aortic disruption? -ANSWER-


Widened mediastinum, obliteration of aortic knob, deviation of trachea to the right,
depression of left mainstem bronchus, deviation of esophagus (NG tube) to right,
widened paratracheal stripe, fx'd 1st/2nd ribs or scapula.

A deceleration injury victim with a left pnuemothorax or hemothorax without rib


fractures, is in pain or shock out of proportion to the apparent injury, and has particulate
matter in their chest tube may have _________. -ANSWER- an ESOPHAGEAL
RUPTURE - a forceful blow causes expulsion of gastric contents into the esophagus,
producing a linear tear in the lower esophagus allowing leakage into the mediastinum.

Fractures for the lower ribs (10-12) should increase suspicion for _____ injury. -
ANSWER- hepatosplenic

Why are upper torso, facial, and arm plethora with petechiae associated with crush
injuries to the chest? -ANSWER- Temporary compression of the superior vena cava.

How does ATLS suggest you should review a chest radiograph? -ANSWER- Trachea &
bronchi, pleural spaces and parenchyma, mediastinum, diaphragm, bones, soft tissues,
tubes & lines.

You should use a size 16 or 18 gauge 6" needle for pericardiocentesis. How do you
insert it? -ANSWER- Puncture the skin 1-2 cm inferior to the left xiphohondral junction
at a 45 degree angle to the skin towards the heart, aiming toward the top of the left
scapula.

What's a good way to know if you've advanced your needle too far during
pericardiocentesis and have entered ventricular muscle? -ANSWER- ECG Changes -
extreme ST-changes, widened QRS, PVCs, etc... Withdrawl needle until ECG returns to
baseline.

What should you do with your needle after you successfully evacuate blood during
pericardiocentesis? -ANSWER- Lock the stopcock and leave the catheter in place in
case it needs to be reevacuated. If possible, use the Seldinger technique to pass a 14
gauge flexible catheter over the guidewire. This is NOT a definitive treatment.
For patients with facial fractures or basillar skull fractures, gastric tubes should be
inserted ____ before doing a DPL. -ANSWER- through the mouth

You need to do retrograde urethrography PRIOR to foley placement if _____. -


ANSWER- inability to void, unstable pelvic fracture, blood at urethral meatus, scrotal
hematoma, perineal ecchymoses, or high-riding prostate.

DPL is considered to be __% sensitive for detecting intraperitoneal bleeding. -


ANSWER- 98

What are the four places you should look first when doing a FAST scan? -ANSWER-
Mediastinum, hepatorenal fossa, splenorenal fossa, pouch of Douglas.

DPL is indicated when a patient with multiple blunt injuries is hemodynamically


unstable, especially when they have _____. -ANSWER- Change in sensorium (brain
injury/EtOH or drug intoxication), change in sensation (spinal cord injury), injury to
adjacent structures (pelvis, lumbar spine), lap-belt sign (from seatbelt), or if patient is
going for long studies (CT, ortho surgery...).

What is the only ABSOLUTE contraindication to DPL? -ANSWER- An existing indication


for laparotomy.

What are some RELATIVE contraindications to DPL? -ANSWER- Morbid obesity,


advanced cirrhosis, preexisting coagulopathy, and previous abdominal operations
(adhesions).

When should you use an open SUPRAUMBILICAL approach for a DPL? -ANSWER-
PELVIC FRACTURES (don't want to enter pelvic hematoma) and ADVANCED
PREGNANCY (don't want to damage enlarged uterus).

When doing a DPL, what INITIAL findings (not from lab) would mandate a laparotomy? -
ANSWER- Free blood (>10 mL) or GI contents (vegetable fiber, bile).

If you don't get gross blood upon initial DPL aspiration, what do you do next for an
adult? For a child? -ANSWER- Adult - 1,000 mL warm isotonic crystalloid. Kid - 10
mL/kg

You've just put a bunch of fluid in the belly and aspirated more fluid for your DPL. No
gross GI contents or anything alarming are present, what QUANTATIVE things would
make the DPL positive? -ANSWER- >100,000 red cells/mm^3, 500 white cells/mm^3, or
BACTERIA (on gram stain).

Your trauma patient needs an urgent laparotomy, can you take them to the CT scanner
first to evaluate injuries? -ANSWER- No, if they need an emergent laparotomy they are
unstable - unstable patients should NOT go to the CT scanner!
What are some indications for laparotomy in patients with penetrating abdominal
wounds? -ANSWER- Unstable, GSW, peritoneal irritation, fascial penetration

What percentage of stab wounds to the anterior abdomen do NOT penetrate the
peritoneum? -ANSWER- 25-33%

Does an early normal serum amylase level exclude major pancreatic trauma? -
ANSWER- NO

Do you need to operate on anyone with an isolated soild organ injury? -ANSWER- No -
not if they remain hemodynamically stable (Of all patients who are initially thought to
havea ISOLATED solid organ injury, <5% will have hollow viscus injury as well).

Which is LESS likely to have a life-threating hemorrhage - an open book or closed book
pelvic fracture? -ANSWER- Closed book - the pelvic volume is compressed, so not as
much room for blood.

Anterior/posterior forces causes _____ book pelvic fractures, and lateral forces cause
_____ book fractures. -ANSWER- AP = Open Book, LATERAL = Closed Book

Which are more common, open or closed book pelvic fracturs? -ANSWER- CLOSED
BOOK - 60-70% (Open book 15-20%, vertical shear 5-15%)

If a patient with a pelvic fracture is positive for intraperitoneal gross blood, a ex-lap is
warranted. What is your next move if that same patient is NEGATIVE for gross
intraperitoneal blood? -ANSWER- Angiography

What do you need to do BEFORE you do a DPL? (Other than getting stuff together and
surgically prepping, etc...) -ANSWER- DECOMPRESS BLADDER, DECOMPRESS
STOMACH

What is "adequate" fluid return when getting DPL fluid back? -ANSWER- 30%

A blown pupil in a patient with a traumatic injury is caused by compression of which


nerve? -ANSWER- Superficial parasympathetic fibers of the CN III (occulomotor).

What is a "normal" ICP in the resting state? -ANSWER- 10mm Hg (Pressures >20,
particularly if sustained, are associated with poor outcomes).

The Monro-Kellie Doctrine describes compensatory mechanisms inside the calvarium to


stabilize pressure - what are the 2 main/first ones? -ANSWER- Venous Blood & CSF
(decreased in equal volumes, when this is exhausted, herniation can occur and brain
perfusion will likely be inadequate).
Patients with a GCS of 3-8 meet the accepted definition of "coma" or "severe brain
injury." What are the GCS scores for "minor" and "moderate" brain injury? -ANSWER-
Minor = 13-15, Moderate = 8-12

When calculating GCS and there is right/left assymetry in the motor response - which
one do you use? -ANSWER- The "BEST" response. (Better predictor than worst
response)

What signs might you see if a patient has a basillar skull fracture? -ANSWER-
PERIORBITAL ECCHYMOSIS (raccoon eyes), RETROAURICULAR ECCHYMOSIS
(Battle sign), and otorrhea/rhinorrhea.

What do you need to know about the GCS? -ANSWER- EVERYTHING - Know it COLD!

What things might require a person with MINOR brain injury get admitted? -ANSWER-
Abnormal CT (or no scan available), penetrating head injury, prolonged LOC, worsening
LOC, moderate to severe HA, significant drug/alcohol intoxication, skull fx,
oto/rhinorrhea, nobody at home to watch, GCS stays <15, focal neuro deficits.

What would you want to do if a patient with a minor brain injury fails to reach a GCS of
15 within 2 hour post injury, had LOC >5 min, are older than 65, emesis x 2, or had
retrograde amnesia >30 minutes? -ANSWER- CT scan - Everything but the 30 min
amnesia makes them HIGH risk for neurosurgical intervention (as would a basillar skull
fx).

What 2 things do you need to do first for everyone with a MODERATE brain injury
(according to ATLS algorithm)? -ANSWER- CT scan, admit to faciolity capable of
definitive neurosurgical care (Moderate = GCS 9-12)

High levels of CO2 will cause cerebral vasculature to _____. -ANSWER- Dilate (to
increase blood flow) - so you might want to HYPERventilate people with brain injuries.

Ideally, you want to wait to perform a GCS on a person with SEVERE brain injury until
what? -ANSWER- BP is normalized

A FAST scan, DPL, or ex-lap should take priority over a CT scan if you can't get the
brain injured patient's BP up to ____ mm Hg. -ANSWER- 100 If a patient has a systolic
over 100 with evidence of intracranial mass (blown pupil, unequal motor exam) THEN a
CT would take first priority.

A midline shift of greater than ___ often indicates the need for neurosurgical evacuation
of the mass/blood. -ANSWER- 5mm

Your patient has a dilated pupil and you want to give mannitol on the way to the CT
scanner or OR. What is the correct dose? -ANSWER- 0.25-1.0 g/kg via rapid bolus
A cast cutter should be removed to remove a trauma victim's helmet if there is evidence
of a c-spine injury or if _____. -ANSWER- the patient experiences pain or paresthesias
during an initial attempt to remove the helmet.

What are the signs of neurogenic shock? -ANSWER- Vasodilation of lower extremity
blood vessels - resulting in pooling of blood and hypotension. This loss of sympathetic
tone may cause bradycardia or inhibit the tachycardic response to hypovolemia.

How do you treat neurogenic shock? -ANSWER- Judicious use of pressors and
MODERATE fluid resuscitation. Too much fluid may result in overload and pulmonary
edema.

What is the most common type of C1 fracture? -ANSWER- Burst fractures (Jefferson
fracture)

What's the difference between types I, II, and III odontoid process fractures? -
ANSWER- I=tip of odontoid, II=fx at base, III=base of odontoid and extends obliquely
into body of axis. (Odontoid process = dens).

What are the indications for c-spine radiographs in a trauma patient? Which x-ray views
should be obtained? -ANSWER- Midline neck pain, tenderness on palpation,
neurological deficits related to c-spine injuries, altered LOC or intoxication. 1) Lateral, 2)
AP, 3) Open mouth odontoid view

With the proper views of the c-spine, and a qualified radiologist - what is the sensitivity
for finding unstable cervical spine injuries? -ANSWER- >97% (CT with 3mm slices
>99%).

Ten percent of all patients with a c-spine fracture have what? -ANSWER- A second,
noncontiguous vertebral column fracture. (So scan the rest of their spine).

Attempts to align the spine for the purpose of immobilization on the backboard are not
recommended if they _______. -ANSWER- cause pain

Can you clear a c-spine without films? -ANSWER- Yes, if they are awake, alert, sober,
neurological normal, have NO pain, and can flex, extend, and move their head to both
sides without pain - you don't need films.

Should a quadriplegic or paraplegic patient be put on a hard board? -ANSWER- Not for
more than 2 hours - get them off ASAP.

What's a big difference in a physical finding between hypovolemic and neurogenic


shock? -ANSWER- Hypovolemic = usually TACHY, Neurogenic = usually BRADY

Partial or total loss of respiratory function may be seen in a patient with a cervical spine
injury above ___. -ANSWER- C6
Why might someone not be able to breathe if they have a long bone fracture? -
ANSWER- Fat embolism - uncommon though

Abnormal arterial blood flow is indicated by an ABI of ____. -ANSWER- <0.9

By LOOKING at the patient, what findings might suggest pelvic injury? -ANSWER- Leg-
length discrepancy, rotation (usually external)

Crush injuries may result in rhabdomyolysis - casts block flow, also iron is released
which forms ROS which then damage cells and impair ability to regulate K+ etc... What
can you do to prevent this? -ANSWER- Volume expansion, and alkalization of urine with
bicarb will reduce intratubular precipitation of myoglobin. UOP should be 100 mL/hr until
myoglobinuria is cleared.

Muscle does not tolerate lack of arterial flow (tourniquet) for more than ___ hours before
necrosis begins. -ANSWER- 6

What things increase the risk for tetanus? -ANSWER- Wounds >6 hours old, wounds
contused or abraded, >1 cm deep, from high velocity missiles, due to burns or cold, and
significantly contaminated wounds.

Should legs be completely straight when splinting? -ANSWER- No, flexion of 10


degrees recommended to take pressure off neurovascular structures.

Any patient with burns covering more than ___% of BSA require fluid resuscitation. -
ANSWER- 20

The palmer surface of a patient's hand represents approximately ___% of their BSA. -
ANSWER- 1%

A high index of suspicion for inhalation injury must be maintained, because patients
may not display clinical evidence for up to ___ hours, by this time edema may prevent
non-surgical intubation. -ANSWER- 24

Carbon monoxide has ____ times the affinity for oxygen as hemoglobin. -ANSWER-
240

Patients with CO levels less than ___% usually don't have any physical symptoms. -
ANSWER- 20%

Adult head BSA = ___%. -ANSWER- 9 (ENTIRE head front and back = 9)

Baby head BSA = __% -ANSWER- 18 (9 front, 9 back)


What is the main difference between adult and baby BSA determination for burns? -
ANSWER- Entire head on baby is 18, whereas it's 9 for adults. This difference of 9 is
made up by the fact that each side (front/back) on adult = 9, but only 7 for kids. (36 vs
28).

Chest BSA = ___%. -ANSWER- 18

Back BSA = ___%. -ANSWER- 18

Arm BSA = ___%. -ANSWER- 9 TOTAL (front AND back).

Leg BSA for adult = ___%. -ANSWER- 18 TOTAL (9 front, 9 back).

Baby front or back of leg BSA =___%. -ANSWER- 7 (TOTAL leg = 14%)

If you add up BSA head, chest, back, arms, and legs you get 99% of BSA. What is the
remaining 1%? -ANSWER- Perineum

Partial/2nd degree burns extend into the _____ whereas full thickness/3rd degree burns
______. -ANSWER- Partial - go into dermis, FULL go all the way through dermis and
into/beyond SQ tissue.

For patients with CO poisoning, the ½ life is ___ when breathing room air and ___
breathing 100% oxygen -ANSWER- 4 hours on RA, 40 min on 100% O2

How do you calculate the Parkland formula? -ANSWER- 4 * weight (kg) * percent BSA
burned = volume in 24 hours (1st half in 8 hrs, 2nd half over 16 hrs).4*70kg*25 percent
= 7 liters in 24 hours. ***Use 25, NOT 0.25)***

Partial or full thickness burns of ___% in patients less than 10 or older than 50 warrants
transfer to a burn center. -ANSWER- 10%

What percent partial/full thickness burns would qualify a 25 year old for a burn center
transfer? -ANSWER- 20%

What anatomical positions with partial/full thickness burns warrant burn center transfer?
-ANSWER- Face, eyes, ears, hands, genitalia, perineum, feet, skin overlying joints.

Does an inhalation injury warrant transfer to a burn center? -ANSWER- YES!!!!!

Should you treat frostbite by soaking body part in water or not? -ANSWER- YES, 40
degree (104F) for 20-30 min should suffice. Don't warm if there is risk of REFREEZING.

Insofar as hypothermia is concerned, patients are not pronounced dead until they are
_____ and dead. -ANSWER- warm
What are you thinking if a child has broken ribs? -ANSWER- MASSIVE force and highly
likely organ damage (since their ribs are very pliable, a huge amount of force is required
to break them, there is often underlying organ damage WITHOUT broken ribs).

How should you insert a Guedel in a kid? -ANSWER- Use tongue blade depressor and
insert gently without turning - otherwise there is great risk for trauma and resultant
hemorrhage. NOT the 180 degree spin trick.

The normal systolic BP in kids can be estimated by what? -ANSWER- 90 mm Hg + (age


x 2)

How do you estimate a child's total circulating volume? -ANSWER- 80 mL/kg

When shock in a child is suspected, how much fluid do you give them? -ANSWER- 20
mL/kg warm crystalloid May need to repeat up to 3 times (60 mL/kg) then consider
blood products.

Optimal UOP for infants is ___ mL/kg/hr. -ANSWER- 2 (1.5 for younger kids, and 1.0 for
older kids).

How much warmed crystalloid should be used for a DPL in kids? -ANSWER- 10 mL/kg
(up to 1000 mL)

What would you see in an infant that would make you suspect very severe brain injury
despite normal LOC? -ANSWER- Bulging fontanelles - these allow tolerance for
expanding masses/swelling...

What is a possible mistake about a blood pressure of 120/80 in a 87 year old man? -
ANSWER- Assuming that normal blood pressure = normovolemia. Many geriatric
patients have uncontrolled hypertension, and if their normal systolic is 180, then 120/80
is relative HYPOtension for them.

How well do geriatric patients do with non-operative management of abdominal injuries


compared to younger people? -ANSWER- Not as well - the risks of non-operative
management are often worse than the risks of surgery.

Why would geriatric patients be MORE susceptible to head bleeds when there is
increased space around a shrinking brain to protect them from contusion? -ANSWER-
Atrophic brains = stretching of the parasagittal bridging veins, making them more prone
to rupture upon impact.

Plasma volume increases during pregnancy, what happens to hematocrit? -ANSWER-


Decreases - dilution by plasma (31-35% is normal in pregnancy)

What would you think of a WBC of 15,000 in a pregnant woman? -ANSWER- Normal, it
can go up to 25,000 during labor!
What should you always assume about a pregnant patient's stomach? -ANSWER- That
it is always full. (Gastric emptying time increases during pregnancy). Early NG tube
placement recommended.

A PaCO2 of 35 to 40 in a pregnant patient may indicate what? -ANSWER- Impending


respiratory failure. It is usually around 30 due to hyperventilation due to increased levels
of progesterone.

True or False: All Rh negative pregnant trauma patients should get Rhogam? -
ANSWER- True, unless the injury is remote from the uterus (distal extremity injury only).
This therapy should be initiated within 72 hours of injury.

When worn correctly, seatbelts reduce fatalities by ___%. -ANSWER- 65-70%, with a
10-fold reduction in serious injury.

A 22-year-old man is hypotensive and tachycardic after a shotgun wound to the left
shoulder. His blood pressure is initially 80/40 mm Hg. After initial fluid
resuscitation his blood pressure increases to 122/84 mm Hg. His heart rate is now
100 beats per minute and his respiratory rate is 28 breaths per minute. A tube
thoracostomy is performed for decreased left chest breath sounds with the return of
a small amount of blood and no air leak. After chest tube insertion, the most
appropriate next step is:
re-examine the chest
A construction worker falls two stories from a building and sustains bilateral
calcaneal fractures. In the emergency department, he is alert, vital signs are
normal, and he is complaining of severe pain in both heels and his lower back.
Lower extremity pulses are strong and there is no other deformity. The suspected
diagnosis is most likely to be confirmed by:
complete spine x-ray series
What is true regarding the initial resuscitation of a trauma patient?
Evidence of improved perfusion after fluid resuscitation could include
improvement in Glasgow Coma Scale score on reevaluation
In managing a patient with a severe traumatic brain injury, the most important
initial step is to:
secure the airway
A previously healthy, 70-kg (154-pound) man suffers an estimated acute blood loss
of 2 liters. What applies to this patient?
An ABG would demonstrate a base deficit between -6 and -10 mEq/L.
The physiological hypervolemia of pregnancy has clinical significance in the
management of the severely injured, gravid woman by:
increasing the volume of blood loss to produce maternal hypotension.
The best assessment of fluid resuscitation of the adult burn patient is:
urinary output of 0.5 mL/kg/hr
The diagnosis of shock must include:
evidence of inadequate organ perfusion
A 7-year-old boy is brought to the emergency department by his parents several
minutes after he fell through a window. He is bleeding profusely from a 6-cm
wound of his medial right thigh. Immediate management of the wound should
consist of:
direct pressure on the wound
For the patient with severe traumatic brain injury, profound hypocarbia should be
avoided to prevent:
cerebral vasoconstriction with diminished perfusion
After being involved in a motor vehicle crash, a 25-year-old man is brought to a
hospital that has surgery capabilities available.. Computed tomography of the chest
and abdomen shows an aortic injury and splenic laceration with free abdominal
fluid. His blood pressure falls to 70 mm Hg after CT. The next step is:
perform an exploratory laparotomy
What statements regarding abdominal trauma in the pregnant patient is TRUE?
Leakage of amniotic fluid is an indication for hospital admission.
The first maneuver to improve oxygenation after chest injury is:
administer supplemental oxygen
A 25-year-old man, injured in a motor vehicular crash, is admitted to the
emergency department. His pupils react sluggishly and his eyes open to pressure.
He does not follow commands, but he does moan periodically. His right arm is
deformed and does not respond to pressure; however, his left hand reaches
purposefully toward the stimulus. Both legs are stiffly extended. His GCS score is:
9
A 20-year-old woman who is at 32 weeks gestation, is stabbed in the upper right
chest. In the emergency department, her blood pressure is 80/60 mm Hg. She is
gasping for breath, extremely anxious, and yelling for help. Breath sounds are
diminished in the right chest. The most appropriate first step is to:
perform needle or finger decompression of the right chest
What findings in an adult is most likely to require immediate management during
the primary survey?
respiratory rate of 40 breaths per minute
The most important, immediate step in the management of an open pneumothorax
is:
placement of an occlusive dressing over the wound
The following are contraindications for tetanus toxoid administration:
history of neurological reaction or severe hypersensitivity to the product
A 56-year-old man is thrown violently against the steering wheel of his truck
during a motor vehicle crash. On arrival in the emergency department he is
diaphoretic and complaining of chest pain. His blood pressure is 60/40 mm Hg and
his respiratory rate is 40 breaths per minute. What best differentiates cardiac
tamponade from tension pneumothorax as the cause of his hypotension?
breath sounds
Bronchial intubation of the right or left mainstem bronchus can easily occur during
infant endotracheal intubation because:
The trachea is relatively short.
A 23-year-old man sustains 4 stab wounds to the upper right chest during an
altercation and is brought by ambulance to a hospital that has full surgical
capabilities. His wounds are all above the nipple. He is endotracheally intubated,
closed tube thoracostomy is performed, fluid resuscitation is initiated through 2
large-caliber IVs. FAST exam does not reveal intraabdominal injuries. His blood
pressure now is 60/0 mm Hg, heart rate is 160 beats per minute, and respiratory
rate is 14 breaths per minute (ventilated with 100% O2). 1500 mL of blood has
drained from the right chest. The most appropriate next step in managing this
patient is to:
urgently transfer the patient to the operating room
A 39-year-old man is admitted to the emergency department after an automobile
collision. He is cyanotic, has insufficient respiratory effort, and has a GCS score of
6. His full beard makes it difficult to fit the oxygen facemask to his face. The most
appropriate next step is to:
restrict cervical motion and attempt orotracheal intubation using 2 people
A patient is brought to the emergency department after a motor vehicle crash. He is
conscious and there is no obvious external trauma. He arrives at the hospital
completely immobilized on a long spine board. His blood pressure is 60/40 mm Hg
and his heart rate is 70 beats per minute. His skin is warm. What do you expect to
see with the patient?
Flaccidity of the lower extremities and loss of deep tendon reflexes are expected.
What is the most effective method for initially treating frostbite?
Moist heat
A 32-year-old man's right leg is trapped beneath his overturned car for nearly 2
hours before he is extricated. On arrival in the emergency department, his right
lower extremity is cool, mottled, insensate, and motionless. Despite normal vital
signs, pulses cannot be palpated below the right femoral artery and the muscles of
the lower extremity are firm and hard. During the management of this patient, what
is most likely to improve the chances for limb salvage?
surgical consultation for right lower extremity fasciotomy
A patient arrives in the emergency department after being beaten about the head
and face with a wooden club. He is comatose and has a palpable depressed skull
fracture. His face is swollen and ecchymotic. He has gurgling respirations and
vomitus on his face and clothing. The most appropriate step after providing
supplemental oxygen and elevating his jaw is to:
suction the oropharynx
A 22-year-old man sustains a gunshot wound to the left chest and is transported to
a small community hospital no surgical capabilities are available. In the emergency
department, a chest tube is inserted and 700 mL of blood is evacuated. The trauma
center accepts the patient in transfer. Just before the patient is placed in the
ambulance for transfer, his blood pressure decreases to 80/68 mm Hg and his heart
rate increases to 136 beats per minute. The next step should be to:
repeat the primary survey and proceed with transfer
A 64-year-old man involved in a high-speed car crash, is resuscitated initially in a
small hospital without surgical capabilities. He has a closed head injury with a
GCS score of 13. He has a widened mediastinum on chest x-ray with fractures of
left ribs 2 through 4, but no pneumothorax. After initiating fluid resuscitation, his
blood pressure is 110/74 mm Hg, heart rate is 100 beats per minute, and respiratory
rate is 18 breaths per minute. He has gross hematuria and a pelvic fracture. You
decide to transfer this patient to a facility capable of providing a higher level of
care. The facility is 128 km (80 miles) away. Before transfer, you should first:
call the receiving hospital and speak to the surgeon on call
Hemorrhage of 20% of the patient's blood volume is associated usually with
tachycardia
What statement concerning intraosseous infusion is TRUE?
Aspiration of bone marrow confirms appropriate positioning of the needle.
A young woman sustains a severe head injury as the result of a motor vehicle
crash. In the emergency department, her GCS is 6. Her blood pressure is 140/90
mm Hg and her heart rate is 80 beats per minute. She is intubated and
mechanically ventilated. Her pupils are 3 mm in size and equally reactive to light.
There is no other apparent injury. The most important principle to follow in the
early management of her head injury is to
avoid hypotension
A 33-year-old woman is involved in a head-on motor vehicle crash. It took 30
minutes to extricate her from the car. Upon arrival in the emergency department,
her heart rate is 120 beats per minute, BP is 90/70 mm Hg, respiratory rate is 16
breaths per minute, and her GCS score is 15. Examination reveals bilaterally equal
breath sounds, anterior chest wall ecchymosis, and distended neck veins. Her
abdomen is flat, soft, and not tender. Her pelvis is stable. Palpable distal pulses are
found in all 4 extremities. Of the following, the most likely diagnosis is:
cardiac tamponade
A hemodynamically normal 10-year-old girl is hospitalized for observation after a
Grade III (moderately severe) splenic injury has been confirmed by computed
tomography (CT). What mandates prompt celiotomy (laparotomy)?
development of peritonitis on physical exam
A 40-year-old woman who was a restrained driver in a motor vehicle crash is
evaluated in the emergency department. She is hemodynamically normal and found
to be paraplegic at the level of T10. What precaution should be taken during
evaluation and management?
Log rolling using 4 people is a safe approach to restrict spinal motion when
moving her.
A trauma patient presents to your emergency department with inspiratory stridor
and a suspected c-spine injury. Oxygen saturation is 88% on high-flow oxygen via
a nonrebreathing mask. The most appropriate next step is to:
restrict cervical motion and establish a definitive airway
When applying the Rule of Nines to infants
The head is proportionally larger in infants than in adults
A healthy young male is brought to the emergency department following a motor
vehicle crash. His vital signs are a blood pressure of 84/60, pulse 123, GCS 10.
The patient moans when his pelvis is palpated. After initiating fluid resuscitation,
the next step in management is:
placement of a pelvic binder
What situations requires Rh immunoglobulin administration to an injured woman?
positive pregnancy test, Rh negative, and has torso trauma
A 22-year-old female athlete is stabbed in her left chest at the third interspace in
the anterior axillary line. On admission to the emergency department and 15
minutes after the incident, she is awake and alert. Her heart rate is 100 beats per
minute, blood pressure 80/60 mm Hg, and respiratory rate 20 breaths per minute. A
chest x-ray reveals a large left hemothorax. A left chest tube is placed with an
immediate return of 1600 mL of blood. The next management step for this patient
is:
prepare for an exploratory thoracotomy
A 6-year-old boy walking across the street is struck by the front bumper of a sports
utility vehicle traveling at 32 kph (20 mph). What's true about this patient?
A pulmonary contusion may be present in the absence of rib fractures.
Adjuncts used during the primary survey
ECG
Pulse ox
CO2 monitoringV
Ventilatory rate
ABGs
Foleys (UOP)
Gastric catheter
FAST or eFAST
DPL
Urinary output is sensitive for
Patient's volume status and renal perfusion
"Golden hour"
The time from injury to definitive care, during which treatment of shock and
traumatic injuries should occur because survival potential is best; also called the
Golden Period.
Leading cause of trauma deaths worldwide
MVCs
Trimodal death distribution
1st: seconds to minutes of injury (apnea)
2nd: minutes to several hours (EDH, SDH, liver lac, pelvic fractures, spleen
ruptures)
3rd: several days to weeks after injury (sepsis and multi-organ failure)
An 18-year-old male was the unrestrained driver in a MVC involving contact with
a tree, He is being transported to the ED by ambulance after a prolonged
extrication process. He is receive oxygen by mask and IVF via one large-bore IV,
and he is immobilized on a long spine board. How would you prepare for arrival of
this patient?
Airway equipment for possible intubation
IV equipment to place a second IV and get blood work
Lab/Xray available
Monitor equipment ready
Notify blood bank and have transfusion protocol available
Consider appropriate transfer
AMPLE hx
Allergies
Medications currently used
Past illnesses/Pregnancy
Last meal
Events/Environment related to the injury
Blunt trauma MOI
Seatbelt use
Steering wheel deofrmation
Presence/activation of airbags
Direction of impact
Damage to vehicle
Patient position
Ejection from vehicle?
Penetrating trauma MOI
Body region
Velocity of weapon
Caliber
Heat loss can occur at moderated temperatures
59 to 68 F (15-20 C)
Prehospital phase should include what interventions and considerations?
Airway maintenance
Breathing support
Control of bleeding and shock
Immobilization
Immediate transport to closest appropriate facility
Hospital preparation for trauma
Resuscitation area
Airway équipement
Warmed IV crystalloid solution
Monitoring devices
Protocol for requesting additional assistance
Transfer agreements
Primary survey
Airway maintenance with restriction of cervical spine motion
Breathing
Circulation
Disability
Exposure/Environmental control
Patients with maxillofacial or head trauma should be presumed to have
A cervical pine injury and cervical spine motion must be restricted
PITFALL: equipment failure
Test regularly
Ensure spare equipment and batteries are readily available
PITFALL: unsuccessful intubation
Identify patients with difficult anatomy
Identify the most experienced/skilled airway manager on team
Ensure appropriate equipment is available
Be prepared to prefer a surgical airway
PITFALL: progressive airway loss
Recognize the dynamic status of the airway
Recognize the injuries that can result in progressive airway loss
Frequently reassess the patient for signs of deterioration of the airway
In a trauma patient with hypotension, what are the two most important causes to
consider in order of importance?
Tension pneumothorax
Hemorrhage
What is the best way to manage rapid external blood loss?
Direct manual pressure on the wound
What are the major areas of internal hemorrhage?
Chest
Abdomen
Retroperitoneum
Pelvis
Long bones
How should fluids be administered in trauma patients with shock?
Warm IVFs
If unresponsive to initial IVF, give blood transfusion immediately
What are the uses for ETCO2?
Detect ROSC
Confirm ET intubation
Help avoid hypoventilation and hyperventilation
You'd like to insert a foley catheter for a trauma patient but you notice urethras
injury. What test should be performed prior to the insertion of a urinary catheter?
Retrograde urethrogram
DDX for blood in gastric aspirate in a trauma patient
Swallowed blood
Traumatic gastric tube placement
UGI injury
What's a C/I to NGT insertion?
Fracture of the cribriform/midface fracture (insert OG instead)
What injuries are at high risk of compartment syndrome in trauma patients?
Long bones
Crush injuries
Circumferential thermal burns
Prolonged ischemia to the limb
What's normal UOP?
Adult: 0.5 ml/kg/hr
Child: 1-2 ml/kg/hr
MIST for obtaining info from EMS
Mechanism and time of injury
Injuries found and suspected
Symptoms and signs
Treatment initiated
Retroperitoneal organs
Abdominal aorta
IVC
Duodeum
Pancreas
Kidneys
Ureters
Posterior aspects of ascending/descending colon
Bladder
Rectum
Reproductive organs
What's the most frequently injured abdominal organ in blunt trauma? Followed by?
Spleen (40-55%)
Liver (35-45%)
Small bowel (5-10%)
Which patients should you consider transferring, and what tests should be
performed prior to transfer?
The patients whose injuries exceed your ability to care for them, either sue to
specialize needs, or resource availably. Only perform testing that enables the
referring physician to resuscitate, stabilize, and ensure the safer transfer of the
patient
What's a pulse oximetry measure?
Oxygen saturation by relative absorption of light by oxyhemoglobin and
deoxyhemoglobin
Gastric catheter placement can induce vomiting
Be prepared to logroll
Ensure suction is immediately available
Special populations that may have physiological responses that do not follow
expected patterns
Children
Pregnant females
Elderly
Obese individuals
Athletes
Why is info about mechanism of injury so important?
The patient's condition is greatly influenced by MOI. It can enhance the
understanding of the patient's condition and anticipated injuries
Possible adjuncts to secondary survey
X-rays of spine and extremities
CT scans of head, chest, abdomen, spine
Contrast urography and angiography
TEE
Bronchoscopy
Esophagoscopy

You might also like